[Układy trójfazowe] Zastosowanie wektorów przestrzennych.

Awatar użytkownika
mdd
Użytkownik
Użytkownik
Posty: 1897
Rejestracja: 14 kwie 2013, o 10:58
Płeć: Mężczyzna
Lokalizacja: Warszawa
Podziękował: 2 razy
Pomógł: 512 razy

[Układy trójfazowe] Zastosowanie wektorów przestrzennych.

Post autor: mdd »

Wektory przestrzenne - podstawowe zależności.
W elektrotechnice w analizie obwodów trójfazowych ma zastosowanie pewna macierz \(\displaystyle{ \textbf{S}}\):
\(\displaystyle{ \textbf{S}=\frac{2}{3}\begin{bmatrix}1&1&1\\1&a&a^2\\1&a^2&a\end{bmatrix} \qquad \leftarrow (1.1)}\)
gdzie:
\(\displaystyle{ a=e^{j120^{\circ}}=-\frac{1}{2}+j\frac{\sqrt{3}}{2}; \qquad a^2=e^{-j120^{\circ}}=-\frac{1}{2}-j\frac{\sqrt{3}}{2} \qquad \leftarrow (1.2)}\)
Stały czynnik \(\displaystyle{ \frac{2}{3}}\) w definicji macierzy \(\displaystyle{ \textbf{S}}\) bywa zastępowany innymi wartościami. Jednak najczęściej w technice napędowej spotkamy się z taką wartością właśnie.

Macierz odwrotna:
\(\displaystyle{ \textbf{S}^{-1}=\frac{1}{2}\begin{bmatrix}1&1&1\\1&a^2&a\\1&a&a^2\end{bmatrix} \qquad \leftarrow (1.3)}\)
Trzy zmienne \(\displaystyle{ x_{A}, x_{B}, x_{C}}\) (prądy, napięcia) w układzie trójfazowym, transformujemy za pomocą macierzy \(\displaystyle{ \textbf{S}}\) do zmiennych \(\displaystyle{ x^{\underline{0}}, x^{I}, x^{II}}\):
\(\displaystyle{ \begin{bmatrix}x^{\underline{0}}\\x^{I}\\x^{II}\end{bmatrix}=\textbf{S}\begin{bmatrix}x_{A}\\x_{B}\\x_{C}\end{bmatrix} \qquad \leftarrow (1.4)}\)
Zmienna \(\displaystyle{ x^{\underline{0}}}\) przyjmuje wartości rzeczywiste, natomiast zmienne \(\displaystyle{ x^{I}, x^{II}}\) ogólnie przyjmują wartości zespolone. Poza tym łatwo pokazać, że \(\displaystyle{ x^{I}= \left( x^{II}\right) ^{*}}\), stąd mamy:
\(\displaystyle{ x^{I}=x_{\alpha}+jx_{\beta}}\)
\(\displaystyle{ x^{II}=x_{\alpha}-jx_{\beta}}\)
Zatem analiza wielkości trójfazowych sprowadza się do rozpatrywania składowej zerowej \(\displaystyle{ x^{\underline{0}}}\) oraz zmiennej zespolonej \(\displaystyle{ x^{I}}\):
\(\displaystyle{ x^{\underline{0}}=\frac{2}{3}\left(x_{A}+x_{B}+x_{C}\right)\\

x^{I}=\frac{2}{3}\left(x_{A}+a \cdot x_{B}+a^{2} \cdot x_{C}\right)=x_{\alpha}+jx_{\beta}}\)
Zazwyczaj w układach trójfazowych nie ma składowej zerowej, tzn. \(\displaystyle{ x_{A}+x_{B}+x_{C}=0}\), stąd zmienne \(\displaystyle{ x_{A}, x_{B}, x_{C}}\) reprezentowane są jednoznacznie przez zmienną \(\displaystyle{ x^{I}}\). Przykładem są prądy przewodowe w układzie trójfazowym bez przewodu neutralnego.

Naturalnie kojarzymy zmienną zespoloną \(\displaystyle{ x^{I}=x_{\alpha}+jx_{\beta}}\) z wektorem na płaszczyźnie, zaczepionym w początku układu współrzędnych. W związku z tym mówi się o reprezentacji trójfazowego układu napięć lub prądów w postaci tzw. wektora przestrzennego.

Dla zmiennych \(\displaystyle{ x_{A}, x_{B}, x_{C}}\) w układzie trójfazowym, takich, że \(\displaystyle{ x_{A}+x_{B}+x_{C}=0}\), wektor przestrzenny je reprezentujący oznaczamy symbolem \(\displaystyle{ \underline{X}}\), czyli:
\(\displaystyle{ \underline{X}=\frac{2}{3}\left(x_{A}+a \cdot x_{B}+a^{2} \cdot x_{C}\right)=X_{\alpha}+jX_{\beta} \qquad \leftarrow (1.5)}\)
Tutaj należy wspomnieć o takich określeniach jak: modulacja wektorowa, falownik wektorowy, sterowanie wektorowe, z którymi w praktyce się można spotkać.
Prędkość wirowania wektora przestrzennego.
Przykładowo, reprezentacja wektorowa symetrycznego trójfazowego układu napięć:
\(\displaystyle{ u_{A}=\sqrt{2}U \cos\left( \omega_{o}t+\psi_{A}\right)\\
u_{B}=\sqrt{2}U \cos\left( \omega_{o}t-120^{\circ}+\psi_{A}\right)\\
u_{C}=\sqrt{2}U \cos\left( \omega_{o}t+120^{\circ}+\psi_{A}\right)}\)
\(\displaystyle{ \underline{U}=\frac{2}{3}\left(u_{A}+a \cdot u_{B}+a^{2} \cdot u_{C}\right)=\sqrt{2}Ue^{j\left( \omega_{o}t+\psi_{A}\right) } \qquad \leftarrow (1.6)}\)
Widać więc, że w tym przypadku wektor przestrzenny napięcia wiruje ze stałą prędkością kątową równą pulsacji napięcia \(\displaystyle{ \omega_{o}}\).

W ogólności prędkość wirowania \(\displaystyle{ \omega_{x}}\) wektora:
\(\displaystyle{ \underline{X}=X_{\alpha}+jX_{\beta}=Xe^{j\psi}; \qquad X=\sqrt{{X_{\alpha}}^{2}+{X_{\beta}}^{2}} \qquad \leftarrow (1.7)}\)
wynosi:
\(\displaystyle{ \omega_{x}=\frac{d \psi}{dt}=\frac{X_{\alpha} \dot{ X_{\beta}}- \dot{ X_{\alpha}}X_{\beta}} {{X_{\alpha}}^{2}+{X_{\beta}}^{2}} \qquad \leftarrow (1.8)}\)
uzasadnienie:    
W przypadku przemiennika częstotliwości (potoczna nazwa: falownik), który na wyjściu wytwarza napięcia \(\displaystyle{ u_{A}, u_{B}, u_{C}}\), takie że: \(\displaystyle{ u_{A}+u_{B}+u_{C}=0}\), częstotliwość wyjściowa \(\displaystyle{ f_{\text{out}}}\) przemiennika jest związana z prędkością \(\displaystyle{ \omega_{u}}\) wirowania wektora przestrzennego \(\displaystyle{ \underline{U}=\frac{2}{3}\left(u_{A}+a \cdot u_{B}+a^{2} \cdot u_{C}\right)}\) zależnością:
\(\displaystyle{ f_{\text{out}}=\frac{\omega_{u}}{2 \pi } \qquad \leftarrow (1.9)}\)
Jeśli rozpatrywać wektor przestrzenny w wirującym układzie współrzędnych, obróconym o kąt \(\displaystyle{ \theta}\) względem nieruchomego układu współrzędnych, to będzie miał on postać:
\(\displaystyle{ \underline{X}_{\theta}=\underline{X}e^{-j\theta} \qquad \leftarrow (1.10)}\)
Czasami równania układu trójfazowego są prostsze jeśli zapiszemy je za pomocą wektorów przestrzennych wyrażonych w wirującym układzie współrzędnych.
Moc chwilowa wyrażona za pomocą wektorów przestrzennych.
Rozważmy moc chwilową trójfazowego źródła napięcia:
\(\displaystyle{ p=\begin{bmatrix}e_{A}&e_{B}&e_{C}\end{bmatrix} \cdot \begin{bmatrix}i_{A}\\i_{B}\\i_{C}\end{bmatrix} \qquad \leftarrow (1.11)}\)
\(\displaystyle{ p=\left( \textbf{S}^{-1} \begin{bmatrix}e^{\underline{0}}\\e^{I}\\e^{II}\end{bmatrix}\right)^{T}\textbf{S}^{-1} \begin{bmatrix}i^{\underline{0}}\\i^{I}\\i^{II}\end{bmatrix}=
\begin{bmatrix}e^{\underline{0}}\\e^{I}\\e^{II}\end{bmatrix}^{T}\left( \textbf{S}^{-1}\right)^{T}\textbf{S}^{-1}\begin{bmatrix}i^{\underline{0}}\\i^{I}\\i^{II}\end{bmatrix} \qquad \leftarrow (1.12)}\)
\(\displaystyle{ \left( \textbf{S}^{-1}\right)^{T}\textbf{S}^{-1}=\frac{1}{2}\begin{bmatrix}1&1&1\\1&a^2&a\\1&a&a^2\end{bmatrix}^{T} \cdot \frac{1}{2}\begin{bmatrix}1&1&1\\1&a^2&a\\1&a&a^2\end{bmatrix}=\frac{1}{4}\begin{bmatrix}3&0&0\\0&0&3\\0&3&0\end{bmatrix} \qquad \leftarrow (1.13)}\)
\(\displaystyle{ p=\frac{3}{4}\begin{bmatrix}e^{\underline{0}}&e^{I}&e^{II}\end{bmatrix}\begin{bmatrix}i^{\underline{0}}\\i^{I}\\i^{II}\end{bmatrix} \qquad \leftarrow (1.14)}\)
Jeśli przyjąć przyjąć, że: \(\displaystyle{ e^{\underline{0}}=0 \wedge i^{\underline{0}}=0}\), to:
\(\displaystyle{ p=\frac{3}{4}\left(e^{I}i^{II}+e^{II}i^{I}\right) \qquad \leftarrow (1.15)}\)
\(\displaystyle{ p=\frac{3}{4}\left( \underline{E} \cdot \underline{I}^{*}+\underline{E}^{*} \cdot \underline{I}\right) \qquad \leftarrow (1.16)}\)
\(\displaystyle{ p=\frac{3}{2}\left( E_{\alpha} I_{\alpha}+E_{\beta} I_{\beta}\right) \qquad \leftarrow (1.17)}\)
\(\displaystyle{ p=\frac{3}{2} \Re\left(\underline{E} \cdot \underline{I}^{*}\right) \qquad \leftarrow (1.18)}\)
Jeśli wyrazić wektory napięcia i prądu w wirującym układzie współrzędnych, to na podstawie zależności (1.10) i (1.18) otrzymamy:
\(\displaystyle{ p=\frac{3}{2} \Re\left(\underline{E}_{\theta} \cdot {\underline{I}_{\theta}}^{*}\right) \qquad \leftarrow (1.19)}\)
W dalszej części tego opracowania zaprezentowane będą przykłady zastosowania wektorów przestrzennych.
Ciąg dalszy nastąpi.
Awatar użytkownika
mdd
Użytkownik
Użytkownik
Posty: 1897
Rejestracja: 14 kwie 2013, o 10:58
Płeć: Mężczyzna
Lokalizacja: Warszawa
Podziękował: 2 razy
Pomógł: 512 razy

[Układy trójfazowe] Zastosowanie wektorów przestrzennych.

Post autor: mdd »

Pole magnetyczne wirujące i wektor przestrzenny prądu.

W trójfazowym silniku elektrycznym o strukturze cylindrycznej mamy następujące podstawowe elementy składowe: stojan i wirnik, pomiędzy którymi jest szczelina powietrzna umożliwiająca ruch wirnika względem stojana. W żłobkach stojana i wirnika są ułożone zezwoje uzwojeń fazowych silnika, a są one ułożone w dość specyficzny sposób. Przyczyną tego jest to, że prąd danej fazy przepływając przez to uzwojenie ma za zadanie wywołać w szczelinie powietrznej pole magnetyczne o rozkładzie sinusoidalnym.
Ukryta treść:    
Zakładamy, że każda z faz \(\displaystyle{ A, B, C}\) silnika elektrycznego o strukturze cylindrycznej wytwarza w szczelinie powietrznej pole magnetyczne o sinusoidalnym rozkładzie (zmienna kątowa \(\displaystyle{ \gamma}\)):
\(\displaystyle{ \begin{cases} B_{A}(\gamma,i_{A})=k \cdot i_{A}\cos\left( p\gamma\right)\\B_{B}(\gamma,i_{B})=k \cdot i_{B}\cos\left( p\gamma-120^{\circ}\right)\\B_{C}(\gamma,i_{C})=k \cdot i_{C}\cos\left( p\gamma+120^{\circ}\right)\end{cases}\qquad \leftarrow (2.1)}\)
gdzie:
\(\displaystyle{ k}\) - stała taka sama dla każdej z faz \(\displaystyle{ A, B, C}\),
\(\displaystyle{ p}\) - liczba par biegunów uzwojenia; \(\displaystyle{ p=1,2,3,...}\)

Korzystając ze związku: \(\displaystyle{ \cos \gamma=\frac{1}{2}\left(e^{j\gamma}+e^{-j\gamma}\right)}\) można powyższe zapisać inaczej:
\(\displaystyle{ \begin{cases} B_{A}(\gamma,i_{A})=\frac{1}{2}k \cdot i_{A}\left(e^{jp\gamma}+e^{-jp\gamma}\right)\\B_{B}(\gamma,i_{B})=\frac{1}{2}k \cdot i_{B}\left(a^{2} \cdot e^{jp\gamma}+a \cdot e^{-jp\gamma}\right)\\B_{C}(\gamma,i_{C})=\frac{1}{2}k \cdot i_{C}\left(a \cdot e^{jp\gamma}+a^{2} \cdot e^{-jp\gamma}\right)\end{cases}\qquad \leftarrow (2.2)}\)
Pole magnetyczne wypadkowe \(\displaystyle{ B(\gamma, i_{A}, i_{B}, i_{C})}\) od wszystkich faz:
\(\displaystyle{ B(\gamma, i_{A}, i_{B}, i_{C})=B_{A}(\gamma,i_{A})+B_{B}(\gamma,i_{B})+B_{C}(\gamma,i_{C})\qquad \leftarrow (2.3)}\)
Na podstawie (2.2), (2.3) oraz definicji wektora przestrzennego (1.5) otrzymujemy:
\(\displaystyle{ B(\gamma, i_{A}, i_{B}, i_{C})=\frac{1}{2}k\underline{I}^{*} \ e^{jp\gamma}+\frac{1}{2}k\underline{I} \ e^{-jp\gamma}=\frac{1}{2}k\left(\underline{I} \ e^{-jp\gamma}\right) ^{*}+\frac{1}{2}k\underline{I} \ e^{-jp\gamma}}\)
Ostatecznie uzyskujemy zależność:
\(\displaystyle{ B(\gamma, i_{A}, i_{B}, i_{C})=k \cdot \Re\left(\underline{I} \ e^{-jp\gamma} \right) \qquad \leftarrow (2.4)}\)
W przypadku gdy:
\(\displaystyle{ \begin{cases}i_{A}=I_{m} \cos\left( \omega_{o}t+\psi_{i}\right)\\ i_{B}=I_{m} \cos\left( \omega_{o}t-120^{\circ}+\psi_{i}\right)\\ i_{C}=I_{m} \cos\left( \omega_{o}t+120^{\circ}+\psi_{i}\right)\end{cases}}\)
to
\(\displaystyle{ \underline{I}=\frac{2}{3}\left(i_{A}+a \cdot i_{B}+a^{2} \cdot i_{C}\right)=I_{m}e^{j\left( \omega_{o}t+\psi_{i}\right) } \qquad \leftarrow (2.5)}\)
Na podstawie (2.4) i (2.5) mamy:
\(\displaystyle{ B(\gamma,t)=k \cdot \Re\left(I_{m}e^{j\left( \omega_{o}t+\psi_{i}\right) } \ e^{-jp\gamma} \right)=kI_{m}\cos\left( \omega_{o}t-p\gamma+\psi_{i}\right) \qquad \leftarrow (2.6)}\)
Otrzymaliśmy równanie fali sinusoidalnej. To tyle na temat zasady powstawania pola wirującego.

Ciąg dalszy nastąpi.

-- 1 maja 2017, o 12:09 --
UWAGA!
Od tej pory wektory przestrzenne będą oznaczane symbolem \(\displaystyle{ \underline{\textbf{X}}}\). Symbol \(\displaystyle{ \underline{X}}\) (stosowany wcześniej) zarezerwowany będzie na oznaczenie zespolonych wartości skutecznych.

Połączenie dwóch trójfazowych źródeł napięcia.

Teraz przeanalizujemy układ połączonych dwóch trójfazowych źródeł napięcia, które przekazują sobie energię. Źródeł takich oczywiście nie można połączyć bezpośrednio. Można to zrobić włączając trójfazowy dławik między te źródła. Przyjmiemy najprostszy model dławika jako trzy liniowe szeregowe dwójniki \(\displaystyle{ RL (\times 3)}\). Układ taki pokazany jest na poniższym rysunku.

Taki układ jest uniwersalny, bo może on być modelem (w pewnym sensie) np. układu: sieć zasilająca - maszyna elektryczna (silnik lub generator), czy też układ sieć zasilająca-przemiennik częstotliwości AFE (ang. active front end) - który w obwodzie wejściowym, zamiast zwykłego prostownika diodowo-tyrystorowego, ma inwerter tranzystorowy, pracujący jako prostownik PWM (w praktyce takie ustrojstwa są coraz częściej używane; jest to tzw. technologia AFE, układ taki umożliwia dwukierunkowy przepływ energii, przy poborze z sieci niemal sinusoidalnego prądu).
Linki do ciekawych stron:    
Równania powyżej przedstawionego obwodu trójfazowego:
\(\displaystyle{ \begin{cases}e_{A}=L\frac{di_{A}}{dt}+Ri_{A}+u_{A}+u_{N}\\e_{B}=L\frac{di_{B}}{dt}+Ri_{B}+u_{B}+u_{N}\\ e_{C}=L\frac{di_{C}}{dt}+Ri_{C}+u_{C}+u_{N}\\i_{A}+i_{B}+i_{C}=0\end{cases}\qquad \leftarrow (3.1)}\)
Dodając stronami pierwsze trzy równania układu (3.1) otrzymujemy:
\(\displaystyle{ e_{A}+e_{B}+e_{C}=L\frac{d}{dt}(i_{A}+i_{B}+i_{C})+R(i_{A}+i_{B}+i_{C})+u_{A}+u_{B}+u_{C}+3u_{N}}\)
Teraz biorąc pod uwagę czwarte równanie układu (3.1) otrzymujemy:
\(\displaystyle{ e_{A}+e_{B}+e_{C}=u_{A}+u_{B}+u_{C} +3u_{N}\qquad \leftarrow (3.2)}\)
\(\displaystyle{ u_{N}}\) - to tzw. napięcie niesymetrii lub niezrównoważenia

Korzystając z (3.1) i z definicji wektora przestrzennego (1.5) otrzymujemy:
\(\displaystyle{ \underline{\textbf{E}}=\frac{2}{3}\left(e_{A}+a \cdot e_{B}+a^{2} \cdot e_{C}\right)=L\frac{d}{dt}\left[\frac{2}{3} \left( i_{A}+a \cdot i_{B}+a^{2} \cdot i_{C}\right)\right]+R \cdot \frac{2}{3}\left( i_{A}+a \cdot i_{B}+a^{2} \cdot i_{C}\right)+\\+\frac{2}{3}\left( u_{A}+a \cdot u_{B}+a^{2} \cdot u_{C}\right)+u_{N}\frac{2}{3}\left(1+a+a^{2}\right)}\)
stąd:
\(\displaystyle{ \underline{\textbf{E}}=L\frac{d\underline{\textbf{I}}}{dt}+R\underline{\textbf{I}} +\underline{\textbf{U}}\qquad \leftarrow (3.3)}\)
Oczywiście:
\(\displaystyle{ \underline{\textbf{I}}=\frac{2}{3}\left( i_{A}+a \cdot i_{B}+a^{2} \cdot i_{C}\right)\qquad \leftarrow (3.4)}\)
\(\displaystyle{ \underline{\textbf{U}}=\frac{2}{3}\left( u_{A}+a \cdot u_{B}+a^{2} \cdot u_{C}\right)\qquad \leftarrow (3.5)}\)
Oczywiście: \(\displaystyle{ 1+a+a^{2}=0}\)

Jeśli rozpatrywać wektory w wirującym układzie współrzędnych:
\(\displaystyle{ \begin{cases} \underline{\textbf{E}}_{dq}=\underline{\textbf{E}}e^{-j\theta}\\ \underline{\textbf{I}}_{dq}=\underline{\textbf{I}}e^{-j\theta}\\ \underline{\textbf{U}}_{dq}=\underline{\textbf{U}}e^{-j\theta}\end{cases} \implies \begin{cases}\underline{\textbf{E}}=\underline{\textbf{E}}_{dq}e^{j\theta}\\ \underline{\textbf{I}}=\underline{\textbf{I}}_{dq}e^{j\theta}\\ \underline{\textbf{U}}=\underline{\textbf{U}}_{dq}e^{j\theta} \end{cases}\qquad \leftarrow (3.6)}\)
to równanie (3.3) przekształci się do postaci:
\(\displaystyle{ \underline{\textbf{E}}_{dq}=L\frac{d\underline{\textbf{I}}_{dq}}{dt}+j\omega L\underline{\textbf{I}}_{dq}+R\underline{\textbf{I}}_{dq} +\underline{\textbf{U}}_{dq}\qquad \leftarrow (3.7)}\)
gdzie:
\(\displaystyle{ \omega=\frac{d \theta}{dt}\qquad \leftarrow (3.8)}\)
Zapiszmy wektory w postaci:
\(\displaystyle{ \underline{\textbf{E}}_{dq}=E_{d}+jE_{q} \qquad \underline{\textbf{I}}_{dq}=I_{d}+jI_{q}\qquad \underline{\textbf{U}}_{dq}=U_{d}+jU_{q}\qquad \leftarrow (3.9)}\)
Wtedy równanie wektorowe (3.7) będzie równoważne układowi równań:
\(\displaystyle{ \begin{cases}E_{d}=L\frac{dI_{d}}{dt}-\omega LI_{q}+RI_{d} +U_{d} \\ E_{q}=L\frac{dI_{q}}{dt}+\omega LI_{d}+RI_{q} +U_{q} \end{cases}\qquad \leftarrow (3.10)}\)
Analogicznie do jednej z metod sterowania prostownika PWM, sterowanie VOC (voltage oriented control) przepływem energii między oboma źródłami trójfazowymi polega na takim wymuszaniu wektora prądu \(\displaystyle{ \underline{\textbf{I}}_{dq}}\) ("za pomocą" wektora \(\displaystyle{ \underline{\textbf{U}}_{dq}}\)), tak żeby kąt pomiędzy wektorami \(\displaystyle{ \underline{\textbf{E}}_{dq}}\) i \(\displaystyle{ \underline{\textbf{I}}_{dq}}\) (a tym samym pomiędzy \(\displaystyle{ \underline{\textbf{E}}}\) i \(\displaystyle{ \underline{\textbf{I}}}\)) miał określoną wartość - najczęściej zero.

Można np. przyjąć układ współrzędnych, tak żeby: \(\displaystyle{ U_{q}=0}\), wtedy \(\displaystyle{ U_{d}=U}\) i układ równań (3.10) przyjmie postać:
\(\displaystyle{ \begin{cases}E_{d}=L\frac{dI_{d}}{dt}-\omega LI_{q}+RI_{d} +U \\ E_{q}=L\frac{dI_{q}}{dt}+\omega LI_{d}+RI_{q} \end{cases}\qquad \leftarrow (3.11)}\)
Wtedy na przebiegi prądów składowych \(\displaystyle{ I_{d}(t), I_{q}(t)}\) mamy wpływ za pomocą \(\displaystyle{ \omega(t)}\) i \(\displaystyle{ U(t)}\).

Połączenie dwóch trójfazowych źródeł napięcia sinusoidalnego - stan ustalony.

W takim stanie \(\displaystyle{ \frac{dI_{d}}{dt}=0 \wedge \frac{dI_{q}}{dt}=0}\) i wszystkie wektory wirują z taką samą prędkością \(\displaystyle{ \omega_{o}}\). Wtedy układ równań (3.10) przyjmie postać:
\(\displaystyle{ \begin{cases}E_{d}=-\omega_{o} LI_{q}+RI_{d} +U_{d} \\ E_{q}=\omega_{o} LI_{d}+RI_{q} +U_{q} \end{cases}\qquad \leftarrow (3.12)}\)
Rozwiązanie powyższego układu:
\(\displaystyle{ I_{d}=\frac{\left( E_{d}-U_{d}\right)R+\omega_{o}L\left( E_{q}-U_{q}\right) }{R^2+\left( \omega_{o}L\right)^{2} } \qquad \leftarrow (3.13)}\)
\(\displaystyle{ I_{q}=\frac{\left( E_{q}-U_{q}\right)R-\omega_{o}L\left( E_{d}-U_{d}\right) }{R^2+\left( \omega_{o}L\right)^{2}} \qquad \leftarrow (3.14)}\)
Dodatkowo wybieramy taki układ współrzędnych, w którym: \(\displaystyle{ E_{q}=0 \wedge E_{d}=E}\) i zakładamy, że \(\displaystyle{ R<<\omega_{o}L}\), inaczej mówiąc przyjmujemy rezystancję \(\displaystyle{ R=0}\). Wtedy wyrażenia (3.13) i (3.14) przyjmują postać:
\(\displaystyle{ I_{d}=-\frac{U_{q}}{ \omega_{o}L } \qquad \leftarrow (3.15)}\)
\(\displaystyle{ I_{q}=\frac{U_{d}-E}{\omega_{o}L} \qquad \leftarrow (3.16)}\)
Ukryta treść:    
Na podstawie powyższych dwóch zależności widać, że aby składowa bierna prądu \(\displaystyle{ I_{q}}\) była równa zeru, to \(\displaystyle{ U_{d}=E}\). Jeśli zaś chcemy by z punktu widzenia źródła \(\displaystyle{ e}\) układ miał charakter pojemnościowy (\(\displaystyle{ I_{q}>0}\); wtedy możemy kompensować moc bierną indukcyjną), to \(\displaystyle{ U_{d}>E}\), a więc ogólnie: \(\displaystyle{ U=\sqrt{{U_{d}}^{2}+{U_{q}}^{2}}>E}\)

Wzajemne położenie dwóch wektorów przestrzennych.

Rozważmy dwa wektory: \(\displaystyle{ \underline{\textbf{X}}_{1}=X_{1}e^{j\psi_{1}}=X_{1\alpha}+jX_{1\beta}, \qquad \underline{\textbf{X}}_{2}=X_{2}e^{j\psi_{2}}=X_{2\alpha}+jX_{2\beta}}\).

Zauważamy, że:
\(\displaystyle{ \underline{\textbf{X}}_{1} \cdot {\underline{\textbf{X}}_{2}}^{*}=X_{1}X_{2}e^{j\left( \psi_{1}-\psi_{2}\right) }=X_{1}X_{2}\cos\left( \psi_{1}-\psi_{2}\right)+jX_{1}X_{2}\sin\left( \psi_{1}-\psi_{2}\right)}\)
\(\displaystyle{ \underline{\textbf{X}}_{1} \cdot {\underline{\textbf{X}}_{2}}^{*}=X_{1\alpha}X_{2\alpha}+X_{1\beta}X_{2\beta}+j\left(X_{1\beta}X_{2\alpha}-X_{1\alpha}X_{2\beta} \right)}\)
\(\displaystyle{ \cos\left( \psi_{1}-\psi_{2}\right)=\frac{\Re\left[ \underline{\textbf{X}}_{1} \cdot \underline{\textbf{X}}_{2}^{*}\right] }{\left| \underline{\textbf{X}}_{1}\right| \cdot \left| \underline{\textbf{X}}_{2}\right| }=\frac{X_{1\alpha}X_{2\alpha}+X_{1\beta}X_{2\beta}}{X_{1}X_{2}} \qquad \leftarrow (3.18)}\)
\(\displaystyle{ \sin\left( \psi_{1}-\psi_{2}\right)=\frac{\Im\left[ \underline{\textbf{X}}_{1} \cdot \underline{\textbf{X}}_{2}^{*}\right] }{\left| \underline{\textbf{X}}_{1}\right| \cdot \left| \underline{\textbf{X}}_{2}\right| }=\frac{X_{1\beta}X_{2\alpha}-X_{1\alpha}X_{2\beta} }{X_{1}X_{2}} \qquad \leftarrow (3.19)}\)
Ukryta treść:    
Awatar użytkownika
mdd
Użytkownik
Użytkownik
Posty: 1897
Rejestracja: 14 kwie 2013, o 10:58
Płeć: Mężczyzna
Lokalizacja: Warszawa
Podziękował: 2 razy
Pomógł: 512 razy

[Układy trójfazowe] Zastosowanie wektorów przestrzennych.

Post autor: mdd »

Wyższe harmoniczne w układach trójfazowych symetrycznych.

Rozważmy trzy prądy fazowe:
\(\displaystyle{ \begin{cases}i_{A}=f(t)\\i_{B}=f\left( t-\frac{T}{3}\right)\\i_{C}=f\left( t+\frac{T}{3}\right)\end{cases}\qquad \leftarrow (4.1)}\)
\(\displaystyle{ f(t)}\) - funkcja o okresie \(\displaystyle{ T}\)

Przedstawiając (4.1) za pomocą szeregu Fouriera mamy:
\(\displaystyle{ i_{A}=\sum_{n \ge 1} I_{m}^{(n)}\cos\left( n \omega_{o}t+\psi^{(n)}\right) \\i_{B}=\sum_{n \ge 1} I_{m}^{(n)}\cos\left( n \omega_{o}t-\frac{2}{3} \pi n+\psi^{(n)}\right)\qquad\qquad \leftarrow (4.2) \\i_{C}=\sum_{n \ge 1} I_{m}^{(n)}\cos\left( n \omega_{o}t+\frac{2}{3} \pi n+\psi^{(n)}\right)}\)
\(\displaystyle{ \omega_{o}=\frac{2 \pi }{T}\qquad \leftarrow (4.3)}\)
\(\displaystyle{ \underline{\textbf{I}}=\frac{2}{3}\left(i_{A}+a \cdot i_{B}+a^{2} \cdot i_{C}\right)=\\=\frac{1}{3}\sum_{n \ge 1}I_{m}^{(n)}\left[ e^{j\left( n \omega_{o}t+\psi^{(n)}\right)}+e^{-j\left( n \omega_{o}t+\psi^{(n)}\right)}\right]+\\
+\frac{1}{3}a\sum_{n \ge 1}I_{m}^{(n)}\left[ e^{j\left( n \omega_{o}t-\frac{2}{3} \pi n+\psi^{(n)}\right)}+e^{-j\left( n \omega_{o}t-\frac{2}{3} \pi n+\psi^{(n)}\right)}\right]+\\
+\frac{1}{3}a^{2}\sum_{n \ge 1}I_{m}^{(n)}\left[ e^{j\left( n \omega_{o}t+\frac{2}{3} \pi n+\psi^{(n)}\right)}+e^{-j\left( n \omega_{o}t+\frac{2}{3} \pi n+\psi^{(n)}\right)}\right]=\\
=\frac{1}{3} \sum_{n \ge 1}\left[I_{m}^{(n)} e^{j\left( n \omega_{o}t+\psi^{(n)}\right)}\left(1+ae^{-j\frac{2}{3} \pi n}+a^{2}e^{j\frac{2}{3} \pi n} \right)\right]+\\+\frac{1}{3} \sum_{n \ge 1}\left[I_{m}^{(n)} e^{-j\left( n \omega_{o}t+\psi^{(n)}\right)}\left( 1+ae^{j\frac{2}{3} \pi n}+a^{2}e^{-j\frac{2}{3} \pi n}\right)\right]=\\
=\frac{1}{3} \sum_{n \ge 1}\left[I_{m}^{(n)} e^{j\left( n \omega_{o}t+\psi^{(n)}\right)}\left(1+e^{-j\frac{2}{3} \pi(n-1) }+e^{j\frac{2}{3} \pi (n-1)} \right)\right]+\\+\frac{1}{3} \sum_{n \ge 1}\left[I_{m}^{(n)} e^{-j\left( n \omega_{o}t+\psi^{(n)}\right)}\left( 1+e^{j\frac{2}{3} \pi (n+1)}+e^{-j\frac{2}{3} \pi (n+1)}\right)\right]}\)
Ostatecznie można zapisać:
\(\displaystyle{ \underline{\textbf{I}}=\sum_{k \ge 1}\left[ I_{m}^{(3k-2)}e^{ j\left( (3k-2) \omega_{o}t+\psi^{(3k-2)}\right)} + I_{m}^{(3k-1)}e^{-j\left( (3k-1) \omega_{o}t+\psi^{(3k-1)}\right)}\right] \qquad \leftarrow (4.4)}\)
Czyli:
\(\displaystyle{ \underline{\textbf{I}}=\\= I_{m}^{(1)}e^{ j\left(\omega_{o}t+\psi^{(1)}\right)} + I_{m}^{(4)}e^{ j\left(4\omega_{o}t+\psi^{(4)}\right)} + I_{m}^{(7)}e^{ j\left(7\omega_{o}t+\psi^{(7)}\right)} + I_{m}^{(10)}e^{ j\left(10\omega_{o}t+\psi^{(10)}\right)} +\ldots+\\
+I_{m}^{(2)}e^{-j\left(2\omega_{o}t+\psi^{(2)}\right)}+I_{m}^{(5)}e^{-j\left(5\omega_{o}t+\psi^{(5)}\right)}+I_{m}^{(8)}e^{-j\left(8\omega_{o}t+\psi^{(8)}\right)}+I_{m}^{(11)}e^{-j\left(11\omega_{o}t+\psi^{(11)}\right)}+\ldots}\)
Jak widać, wektor przestrzenny dla harmonicznych rzędu \(\displaystyle{ 1,4,7,10,\ldots}\) wiruje w przeciwną stronę niż wektor przestrzenny dla harmonicznych rzędu \(\displaystyle{ 2,5,8,11, \ldots}\). Powyższe rozważanie jest szczególnie istotne jeśli chodzi o wpływ poszczególnych harmonicznych prądu na magnetyczne pole wirujące w maszynie elektrycznej oraz na wytwarzany moment elektromagnetyczny.

W trójprzewodowym symetrycznym układzie trójfazowym mogą wystąpić harmoniczne prądów o rzędach:
1) \(\displaystyle{ (3k-2); k=1,2,3, \ \ldots}\) - składowe zgodne;
2) \(\displaystyle{ (3k-1); k=1,2,3, \ \ldots}\) - składowe przeciwne.

Harmoniczne o rzędach \(\displaystyle{ 3k, \ k=1,2,3,\dots}\) w symetrycznym układzie trójprzewodowym w ogóle nie występują, ze względu na warunek \(\displaystyle{ i_{A}+i_{B}+i_{C}=0}\).

Jeśli wziąć pod uwagę to, że w praktyce przebiegi są antysymetryczne, to można wykluczyć również harmoniczne parzyste. W związku z tym rzędy wyższych harmonicznych prądów w układzie trójprzewodowym: \(\displaystyle{ (6k \pm 1); \ k=1,2,3,\ldots}\).

Jeśli rozważyć wyrażenie \(\displaystyle{ \left( 1 \pm 6k\right) ; \ k=1,2,3,\ldots}\), to wartość bezwzględna tego wyrażenia będzie rzędem harmonicznej, zaś znak tego wyrażenia będzie informował nas o tym, czy harmoniczna rzędu \(\displaystyle{ \left| 1 \pm 6k\right|; \ k=1,2,3, \ldots}\) jest składową zgodną, czy składową przeciwną.

Równania silnika PMSM.

PMSM - ang. permanent magnet synchronous motor, czyli silnik synchroniczny z magnesami trwałymi.
Silniki te mają wiele zalet, między innymi: duża sprawność, małe rozmiary w stosunku do silników indukcyjnych klatkowych. Wadą jest wysoka cena (ze względu na koszt wytwarzania magnesów trwałych) oraz to, że silniki PMSM mają mały moment rozruchowy (np. w porównaniu do indukcyjnych silników klatkowych), w związku z tym konieczne jest stosowanie rozruchu częstotliwościowego (oczywiście na rynku spotykane są również silniki PMSM z klatkami rozruchowymi, dzięki którym silniki uzyskują duży moment rozruchowy, wtedy rozruch silnika odbywa się tak jak w zwykłym silniku klatkowym, a po osiągnięciu przez silnik synchronizmu, klatki rozruchowe nie mają wpływu na pracę silnika - przynajmniej w stanie ustalonym). Ze względu na rozpowszechnienie przemienników częstotliwości, które pierwotnie były dedykowane głównie do współpracy z silnikami klatkowymi, a które obecnie coraz częściej mają możliwość współpracy z silnikami synchronicznymi (PMSM lub silnikami reluktancyjnymi), silniki PMSM są coraz częściej stosowane (raczej nie prędko będą tak popularne jak zwykłe "klatkowce"). Silniki PMSM są stosowane zazwyczaj tam, gdzie wymagane jest precyzyjne sterowanie prędkości i położenia. Silniki PMSM z enkoderami absolutnymi współpracujące z przemiennikami częstotliwości (serwonapędy), mają możliwość bardzo precyzyjnego sterowania momentem elektromagnetycznym, przede wszystkim w zakresie bardzo małych prędkości (tutaj silnik PMSM ma sporą przewagę nad indukcyjnym silnikiem klatkowym). Należy sobie uświadomić, że silnik PMSM jest cały czas namagnesowany i gotowy do wytworzenia momentu (w odróżnieniu od silnika indukcyjnego, którego najpierw trzeba namagnesować). Pole magnetyczne wirnika silnika PMSM jest nieruchome względem wirnika, natomiast w silniku indukcyjnym występuje zawsze pewien ruch względny pomiędzy polem wirnika, a samym wirnikiem.
\(\displaystyle{ \begin{bmatrix}u_{A}\\u_{B}\\u_{C}\end{bmatrix}=\begin{bmatrix}R&0&0\\0&R&0\\0&0&R\end{bmatrix}\begin{bmatrix}i_{A}\\i_{B}\\i_{C}\end{bmatrix}+\frac{d}{dt}\begin{bmatrix}L_{\sigma s}+L_{o}&-\frac{1}{2}L_{o}&-\frac{1}{2}L_{o}\\-\frac{1}{2}L_{o}&L_{\sigma s}+L_{o}&-\frac{1}{2}L_{o}\\-\frac{1}{2}L_{o}&-\frac{1}{2}L_{o}&L_{\sigma s}+L_{o}\end{bmatrix}\begin{bmatrix}i_{A}\\i_{B}\\i_{C}\end{bmatrix}+\\+L\frac{d}{dt}\begin{bmatrix}\cos 2p \gamma & \cos \left( 2p \gamma-120^{\circ}\right) & \cos \left( 2p\gamma+120^{\circ}\right)\\ \cos \left( 2p\gamma-120^{\circ}\right) &\cos \left( 2p \gamma+120^{\circ}\right)&\cos 2p \gamma\\ \cos \left( 2p\gamma+120^{\circ}\right)& \cos 2p \gamma & \cos \left( 2p \gamma-120^{\circ}\right)\end{bmatrix}\begin{bmatrix}i_{A}\\i_{B}\\i_{C}\end{bmatrix}+\\+\Psi_{o}\frac{d}{dt}\begin{bmatrix}\cos p \gamma\\ \cos \left( p \gamma-120^{\circ}\right) \\ \cos \left( p \gamma+120^{\circ}\right) \end{bmatrix} \qquad \leftarrow (5.1)}\)
\(\displaystyle{ \gamma}\) - kąt określający położenie wirnika względem stojana;
\(\displaystyle{ p}\) - liczba par biegunów uzwojenia stojana;
\(\displaystyle{ R}\) - rezystancja uzwojenia stojana;
\(\displaystyle{ \Psi_{o}}\) - amplituda strumienia skojarzonego z uzwojeniem stojana i wytworzonego przez magnesy trwałe umieszczone w wirniku silnika;
\(\displaystyle{ L_{\sigma s}}\) - indukcyjność rozproszenia uzwojenia stojana.

Jak widać indukcyjność uzwojeń stojana ma dwie składowe: stałą oraz zmienną o amplitudzie \(\displaystyle{ L}\) (zależną od wzajemnego usytuowania wirnika względem stojana, opisanego poprzez kąt \(\displaystyle{ \gamma}\)).

Równanie (5.1) można zapisać bardziej zwięźle:
\(\displaystyle{ \textbf{u}=\textbf{R}\textbf{i}+\frac{d}{dt}\left( \textbf{L}_{s}\textbf{i}\right)+\frac{d}{dt}\left( \textbf{L}_{r}\textbf{i}\right)+\frac{d}{dt}\mathbf{\Psi} \qquad \leftarrow (5.2)}\)
Mnożąc lewostronnie równanie (5.1) przez macierz \(\displaystyle{ \textbf{S}}\) (patrz wzór (1.1)) otrzymujemy:
\(\displaystyle{ \textbf{S}\textbf{u}=\textbf{S}\textbf{R}\textbf{i}+\frac{d}{dt}\left( \textbf{S} \textbf{L}_{s}\textbf{i}\right)+\frac{d}{dt}\left(\textbf{S} \textbf{L}_{r}\textbf{i}\right)+\frac{d}{dt}\left( \textbf{S}\mathbf{\Psi}\right) \qquad \leftarrow (5.3)}\)
Teraz jeszcze macierz prądów \(\displaystyle{ \textbf{i}}\) i napięć \(\displaystyle{ \textbf{u}}\) wyrażamy przez macierz składowych \(\displaystyle{ \underline{0}, I, II}\):
\(\displaystyle{ \textbf{i}=\textbf{S}^{-1} \textbf{i}^{+}; \qquad \textbf{u}^{+}=\textbf{S} \textbf{u}; \qquad \mathbf{\Psi}^{+}=\textbf{S} \mathbf{\Psi}\qquad \leftarrow (5.4)}\)
\(\displaystyle{ \textbf{u}^{+}=\textbf{S}\textbf{R}\textbf{S}^{-1} \textbf{i}^{+}+\frac{d}{dt}\left( \textbf{S} \textbf{L}_{s}\textbf{S}^{-1}\textbf{i}^{+}\right)+\frac{d}{dt}\left(\textbf{S} \textbf{L}_{r}\textbf{S}^{-1}\textbf{i}^{+}\right)+\frac{d}{dt}\left( \mathbf{\Psi}^{+}\right) \qquad \leftarrow (5.5)}\)
Teraz wykonujemy przekształcenia macierzy::    
Ostatecznie równania silnika PMSM napiszemy w postaci:
\(\displaystyle{ u^{\underline{0}}=Ri^{\underline{0}}+L_{\sigma s}\frac{d i^{\underline{0}}}{dt} \\ u^{I}=Ri^{I}+\left( L_{\sigma s}+\frac{3}{2}L_{o}\right) \frac{d i^{I}}{dt}+\frac{3}{2}L\frac{d}{dt}\left( i^{II}e^{j2p \gamma}\right)+\frac{d}{dt}\left( \Psi_{o}e^{jp \gamma}\right)
\\u^{II}=Ri^{II}+\left( L_{\sigma s}+\frac{3}{2}L_{o}\right) \frac{d i^{II}}{dt}+\frac{3}{2}L\frac{d}{dt}\left( i^{I}e^{-j2p \gamma}\right)+\frac{d}{dt}\left( \Psi_{o}e^{-jp \gamma}\right)}\)
W związku z tym, że składowe \(\displaystyle{ I}\) i \(\displaystyle{ II}\) są wzajemnie sprzężone oraz tym, że składowa zerowa prądów "żyje własnym życiem" (wcześniejsze rozważania pokazały, że nie ma ona wpływu na pole magnetyczne w silniku - przynajmniej w naszym prostym modelu), w dalszym ciągu rozważań bierzemy pod uwagę tylko równanie dla składowej \(\displaystyle{ I}\). Oznaczając składową \(\displaystyle{ I}\) symbolem wektora przestrzennego otrzymujemy:
\(\displaystyle{ \underline{\textbf{U}}=R\underline{\textbf{I}}+\left( L_{\sigma s}+\frac{3}{2}L_{o}\right) \frac{d \underline{\textbf{I}}}{dt}+\frac{3}{2}L \frac{d}{dt}\left( \underline{\textbf{I}}^{*}e^{j2p \gamma}\right)+\frac{d}{dt}\left( \Psi_{o}e^{jp \gamma}\right) \qquad \leftarrow (5.6)}\)
Szczególnie elegancką postać przyjmują równania silnika PMSM przy użyciu wektorów napięcia i prądu widzianych z punktu widzenia wirującego układu współrzędnych:
\(\displaystyle{ \underline{\textbf{U}}=\underline{\textbf{U}}_{dq}e^{jp \gamma}; \qquad \underline{\textbf{I}}=\underline{\textbf{I}}_{dq}e^{jp \gamma}\qquad \leftarrow (5.7)}\)
\(\displaystyle{ \underline{\textbf{U}}_{dq}e^{jp \gamma}=R\underline{\textbf{I}}_{dq}e^{jp \gamma}+\left( L_{\sigma s}+\frac{3}{2}L_{o}\right) \frac{d}{dt}\left( \underline{\textbf{I}}_{dq}e^{jp \gamma}\right) +\frac{3}{2}L \frac{d}{dt}\left( \underline{\textbf{I}}_{dq}^{*}e^{jp \gamma}\right)+\frac{d}{dt}\left( \Psi_{o}e^{jp \gamma}\right)}\)
Ostatecznie:
\(\displaystyle{ \underline{\textbf{U}}_{dq}=R\underline{\textbf{I}}_{dq}+\left( L_{\sigma s}+\frac{3}{2}L_{o}\right) \left( \frac{d\underline{\textbf{I}}_{dq}}{dt}+jp\omega \underline{\textbf{I}}_{dq}\right)+\\+\frac{3}{2}L \left( \frac{d\underline{\textbf{I}}_{dq}^{*}}{dt}+jp \omega \underline{\textbf{I}}_{dq}^{*}\right) + jp \omega \Psi_{o}\quad \leftarrow (5.8)}\)
\(\displaystyle{ \omega=\frac{d \gamma}{dt}\qquad \leftarrow (5.9)}\)
Po rozbiciu wektorów prądu i napięcia na część rzeczywistą i urojoną w (5.8): \(\displaystyle{ \underline{\textbf{U}}_{dq}=U_{d}+jU_{q}; \ \underline{\textbf{I}}_{dq}=I_{d}+jI_{q}}\) otrzymujemy układ równań:
\(\displaystyle{ \begin{cases}U_{d}=RI_{d}+\frac{d \Psi_{d}}{dt}-p\omega \Psi_{q} \\U_{q}=RI_{q}+\frac{d \Psi_{q}}{dt}+p\omega \Psi_{d} \\\Psi_{d}=L_{d}I_{d}+\Psi_{o} \\ \Psi_{q}=L_{q}I_{q} \\L_{d}=L_{\sigma s}+\frac{3}{2}L_{o}+\frac{3}{2}L\\ L_{q}=L_{\sigma s}+\frac{3}{2}L_{o}-\frac{3}{2}L\end{cases}}\)\(\displaystyle{ \qquad \leftarrow (5.10)}\)
Pozostaje jeszcze znaleźć wyrażenie na moment elektromagnetyczny \(\displaystyle{ T_{e}}\) silnika. W tym celu wykorzystujemy bilans energetyczny. W literaturze można spotkać wyprowadzenie wzoru na moment silnika przy wykorzystaniu energii czy koenergii pola magnetycznego. Tutaj pokażę drogę "na skróty". Moc chwilowa trójfazowego źródła napięcia zasilającego silnik na podstawie (1.14) i (1.19), oraz przy wykorzystaniu pierwszych dwóch równań z układu (5.10):
\(\displaystyle{ P=\frac{3}{4}u^{\underline{0}}i^{\underline{0}}+\frac{3}{2}\left(U_{d}I_{d}+U_{q}I_{q}\right)=\\=\frac{3}{4}u^{\underline{0}}i^{\underline{0}}+\frac{3}{2} \left( R{I_{d}}^{2}+ I_{d} \frac{d \Psi_{d}}{dt}+R{I_{q}}^{2}+I_{q}\frac{d \Psi_{q}}{dt}\right) +\blue{\frac{3}{2}p\omega \left( \Psi_{d}I_{q}-\Psi_{q}I_{d}\right)}}\)
Wyrażenie zaznaczone na niebiesko od razu kojarzymy ze wzorem z mechaniki \(\displaystyle{ P=\omega \cdot T}\), stąd wyrażenie na moment elektromagnetyczny silnika:
\(\displaystyle{ T_{e}=\frac{3}{2}p \left( \Psi_{d}I_{q}-\Psi_{q}I_{d}\right)\quad \leftarrow (5.11)}\)
Powyższy wzór można też zapisać tylko przy użyciu składowych wektora prądu:
\(\displaystyle{ T_{e}=\frac{3}{2}p \left( L_{d}-L_{q}\right)I_{d}I_{q}+\frac{3}{2}p\Psi_{o}I_{q} \quad \leftarrow (5.12)}\)
We wzorze (5.12) wyraźnie widać wyodrębnioną tzw. składową reluktancyjną (pierwszy składnik) momentu silnika.

Warto zwrócić uwagę na jeszcze jeden sposób zapisu wzoru (5.11):
\(\displaystyle{ T_{e}=\frac{3}{2}p \left| \mathbf{\underline{\Psi}}\right| \cdot \left| \underline{\textbf{I}} \right| \cdot \sin \angle \left(\mathbf{\underline{\textbf{I}},\underline{\Psi}} \right) \quad \leftarrow (5.13)}\)
Oczywiście: \(\displaystyle{ \mathbf{\underline{\Psi}}=\Psi_{d}+j\Psi_{q}}\).

O wektorze przestrzennym jako reprezentacji trzech wielkości fazowych bez składowej zerowej.

Rozważmy dowolny wektor przestrzenny:
\(\displaystyle{ \underline{\textbf{X}}=X_{\alpha}+jX_{\beta}\qquad \leftarrow (6.1)}\)
oraz oś \(\displaystyle{ k}\), której położenie opiszemy wektorem jednostkowym: \(\displaystyle{ \underline{\textbf{k}}=e^{j\theta}}\). Wektor przestrzenny \(\displaystyle{ \underline{\textbf{X}}}\) rozkładamy w następujący sposób:
\(\displaystyle{ \underline{\textbf{X}}=X_{\alpha}+jX_{\beta}=X_{k}\underline{\textbf{k}}+jX_{k}^{\perp}\underline{\textbf{k}}\qquad \leftarrow (6.2)}\)
\(\displaystyle{ X_{k}\underline{\textbf{k}}}\) - to rzut wektora \(\displaystyle{ \underline{\textbf{X}}}\) na oś wyznaczoną przez wektor jednostkowy \(\displaystyle{ \underline{\textbf{k}}}\).

Na podstawie (6.2) i definicji wektora jednostkowego \(\displaystyle{ \underline{\textbf{k}}=e^{j\theta}}\), można pokazać, że:
\(\displaystyle{ X_{k}=X_{\alpha}\cos \theta -X_{\beta}\sin \theta=\Re\left\{ \underline{\textbf{k}} \cdot \underline{\textbf{X}}\right\} \qquad \leftarrow (6.3)}\)
Rozważmy teraz rzuty wektora \(\displaystyle{ \underline{\textbf{X}}}\) na osie wyznaczone przez wektory jednostkowe:
\(\displaystyle{ \underline{\textbf{k}}_{1}=e^{j0^{\circ}}=1; \quad \underline{\textbf{k}}_{2}=a^{2}=e^{-j120^{\circ}}; \quad \underline{\textbf{k}}_{3}=a=e^{j120^{\circ}} \qquad \leftarrow (6.4)}\)
Na podstawie (6.1), (6.3) i (6.4) otrzymujemy:
\(\displaystyle{ X_{k1}=X_{\alpha}\\ X_{k2}=-\frac{1}{2}X_{\alpha}+\frac{\sqrt{3}}{2}X_{\beta} \qquad \ \leftarrow (6.5)\\ X_{k3}=-\frac{1}{2}X_{\alpha}-\frac{\sqrt{3}}{2}X_{\beta}}\)
Zauważamy, że: \(\displaystyle{ X_{k1}+X_{k2}+X_{k3}=0}\).

Wróćmy do wektora przestrzennego jako reprezentacji trzech wielkości fazowych: \(\displaystyle{ \underline{\textbf{X}}=\frac{2}{3}\left(x_{A}+a \cdot x_{B}+a^{2} \cdot x_{C}\right)}\). Póki co nie czynimy żadnego dodatkowego założenia odnośnie wielkości \(\displaystyle{ x_{A}, x_{B}, x_{C}}\). Czyli w ogólności:
\(\displaystyle{ X_{\alpha}=\Re \underline{\textbf{X}}=\frac{1}{3}\left( 2x_{A}-x_{B}-x_{C}\right), \ \ X_{\beta}= \Im \underline{\textbf{X}}=\frac{1}{\sqrt{3}}\left( x_{B}-x_{C}\right) \qquad \leftarrow (6.6)}\)
Na podstawie (6.5) i (6.6) mamy:
\(\displaystyle{ X_{k1}=x_{A}-\frac{1}{3}\left( x_{A}+x_{B}+x_{C}\right) \\ \\
X_{k2}=x_{B}-\frac{1}{3}\left( x_{A}+x_{B}+x_{C}\right) \qquad \leftarrow (6.7)\\ \\ X_{k3}=x_{C}-\frac{1}{3}\left( x_{A}+x_{B}+x_{C}\right)}\)
Porównując (6.5) i (6.7) otrzymujemy:
\(\displaystyle{ \begin{cases}x_{A}=X_{\alpha}+X_{0} \\ x_{B}=-\frac{1}{2}X_{\alpha}+\frac{\sqrt{3}}{2}X_{\beta}+X_{0} \\ x_{C}=-\frac{1}{2}X_{\alpha}-\frac{\sqrt{3}}{2}X_{\beta}+X_{0}\end{cases}\qquad \leftarrow (6.8)}\)
gdzie:
\(\displaystyle{ X_{0}=\frac{1}{3}\left( x_{A}+x_{B}+x_{C}\right) \qquad \leftarrow (6.9)}\)
to tzw. składowa zerowa. Tutaj widzimy, że sam wektor przestrzenny \(\displaystyle{ \underline{\textbf{X}}}\) nie określa jednoznacznie trzech wielkości fazowych \(\displaystyle{ x_{A}, x_{B}, x_{C}}\). Potrzebna jest jeszcze informacja o składowej zerowej zdefiniowanej jako średnia arytmetyczna wielkości fazowych.

W literaturze mówi się o tzw. składowych \(\displaystyle{ \alpha, \beta, 0}\) (albo o składowych \(\displaystyle{ \alpha, \beta, \gamma}\)) (https://en.wikipedia.org/wiki/Alpha%E2%80%93beta_transformation). Na podstawie (6.8) otrzymuje się równania:
\(\displaystyle{ \begin{cases}X_{0}=\frac{1}{3}\left( x_{A}+x_{B}+x_{C}\right)\\
X_{\alpha}=\frac{1}{3}\left( 2x_{A}-x_{B}-x_{C}\right)\\
X_{\beta}=\frac{1}{\sqrt{3}}\left( x_{B}-x_{C}\right)\end{cases} \qquad \leftarrow (6.10)}\)
Równania (6.10) stanowią tzw. transformację Edyty Clarke (https://en.wikipedia.org/wiki/Edith_Clarke).

Jeśli założyć, że wielkości fazowe nie mają składowej zerowej, tzn.:
\(\displaystyle{ x_{A}+x_{B}+x_{C}=0 \qquad \leftarrow (6.11)}\)
to na podstawie powyższego i (6.7) otrzymujemy:
\(\displaystyle{ X_{k1}=x_{A} \\ X_{k2}=x_{B} \qquad \leftarrow (6.12) \\ X_{k3}=x_{C}}\)
Widać więc bardzo ciekawy i prosty związek między wektorem przestrzennym będącym reprezentacją trzech wielkości fazowych bez składowej zerowej, a rzutami tego wektora na trzy osie wyznaczone przez wektory \(\displaystyle{ \underline{\textbf{k}}_{1}=1, \underline{\textbf{k}}_{2}=a^{2},\underline{\textbf{k}}_{3}=a}\).

Na podstawie (6.12) i (6.5) mamy związek:
\(\displaystyle{ \left| \underline{\textbf{X}}\right|=\sqrt{\frac{2}{3}\left( {x_{A}}^{2}+{x_{B}}^{2}+{x_{C}}^{2}\right) } \qquad \leftarrow (6.13)}\)
Przy czym należy podkreślić, że zależność ta obowiązuje tylko w przypadku gdy: \(\displaystyle{ x_{A}+x_{B}+x_{C}=0}\).

Praca silnika PMSM w stanie ustalonym.

Zakładamy następujące prądy i wymuszające je napięcia:
\(\displaystyle{ \begin {cases}i_{A}=\sqrt{2}I\cos\left( \omega_{o}t+\psi_{i}\right)\\ i_{B}=\sqrt{2}I\cos\left( \omega_{o}t-120^{\circ}+\psi_{i}\right)\\ i_{C}=\sqrt{2}I\cos\left( \omega_{o}t+120^{\circ}+\psi_{i}\right) \end{cases} \quad \begin{cases}u_{A}=\sqrt{2}U \cos\left( \omega_{o}t+\psi_{A}\right)\\ u_{B}=\sqrt{2}U \cos\left( \omega_{o}t-120^{\circ}+\psi_{A}\right)\\ u_{C}=\sqrt{2}U \cos\left( \omega_{o}t+120^{\circ}+\psi_{A}\right)\end{cases}}\)
Stąd wektory przestrzenne je reprezentujące:
\(\displaystyle{ \underline{\textbf{U}}=\frac{2}{3}\left(u_{A}+a \cdot u_{B}+a^{2} \cdot u_{C}\right)=\sqrt{2}Ue^{j\left( \omega_{o}t+\psi_{A}\right) }=\sqrt{2}\underline{U}e^{j\omega_{o}t} \qquad \leftarrow (7.1)}\)
\(\displaystyle{ \underline{\textbf{I}}=\frac{2}{3}\left(i_{A}+a \cdot i_{B}+a^{2} \cdot i_{C}\right)=\sqrt{2}Ie^{j\left( \omega_{o}t+\psi_{i}\right)}=\sqrt{2}\underline{I}e^{j\omega_{o}t} \qquad \leftarrow (7.2)}\)
gdzie:
\(\displaystyle{ \underline{U}=Ue^{j\psi_{u}}, \quad \underline{I}=Ie^{j\psi_{i}} \qquad \leftarrow (7.3)}\)
Wektory prądu \(\displaystyle{ \underline{\textbf{I}}}\) i napięcia \(\displaystyle{ \underline{\textbf{U}}}\) transformujemy do układu wirującego:
\(\displaystyle{ \underline{\textbf{U}}_{dq}=\underline{\textbf{U}}e^{-jp \gamma}; \qquad \underline{\textbf{I}}_{dq}=\underline{\textbf{I}}e^{-jp \gamma}\qquad \leftarrow (7.4)}\)
W stanie ustalonym wektory prądu \(\displaystyle{ \underline{\textbf{I}}}\) i napięcia \(\displaystyle{ \underline{\textbf{U}}}\) wirują synchronicznie z prędkością kątową \(\displaystyle{ \omega_{o}=p\omega}\) (patrz wzór (2.6)).
Stąd na podstawie (5.9) stwierdzamy, że:
\(\displaystyle{ \omega=\frac{d \gamma}{dt}=\frac{\omega_{o}}{p} \implies \gamma=\frac{\omega_{o}}{p}t+\gamma_{o}\qquad \leftarrow (7.5)}\)
Podstawiając (7.1), (7.2) i (7.5) do (7.4) otrzymujemy:
\(\displaystyle{ \underline{\textbf{U}}_{dq}=\sqrt{2}\underline{U}e^{-jp\gamma_{o}} \qquad \leftarrow (7.6)}\)
\(\displaystyle{ \underline{\textbf{I}}_{dq}=\sqrt{2}\underline{I}e^{-jp\gamma_{o}}\qquad \leftarrow (7.7)}\)
Dla uproszczenia obliczeń przyjmujemy \(\displaystyle{ \psi_{u}=0}\), stąd z powyższego i z (7.6) wynika, że:
\(\displaystyle{ \begin{cases}U_{d}=\Re \underline{\textbf{U}}_{dq}=\sqrt{2}U \cos\left(p\gamma_{o}\right)\\
U_{q}=\Im \underline{\textbf{U}}_{dq}=-\sqrt{2}U \sin\left(p\gamma_{o}\right)\\ \end{cases}\qquad \leftarrow (7.8)}\)
Na podstawie (5.10) i (7.8) mamy:
\(\displaystyle{ \begin{cases}\sqrt{2}U \cos\left(p\gamma_{o}\right)=RI_{d}-X_{q}I_{q} \\-\sqrt{2}U \sin\left(p\gamma_{o}\right)=RI_{q}+X_{d}I_{d}+\omega_{o}\Psi_{o} \\ T_{e}=\frac{3}{2}p \left( L_{d}-L_{q}\right)I_{d}I_{q}+\frac{3}{2}p\Psi_{o}I_{q}\end{cases}\qquad \leftarrow (7.9)}\)
\(\displaystyle{ X_{d}=\omega_{o}L_{d}, \qquad X_{q}=\omega_{o}L_{q}\qquad \leftarrow (7.10)}\)
W pierwszym przybliżeniu zakładamy, że \(\displaystyle{ R=0}\), wtedy z (7.9) wynika, że:
\(\displaystyle{ I_{q}=-\frac{\sqrt{2}U \cos\left(p\gamma_{o}\right)}{X_{q}} \qquad \leftarrow (7.11)}\)
\(\displaystyle{ I_{d}=\frac{-\sqrt{2}U \sin\left(p\gamma_{o}\right)-\omega_{o}\Psi_{o}}{X_{d}} \qquad \leftarrow (7.12)}\)
\(\displaystyle{ T_{e}=\frac{3}{2}\frac{p}{\omega_{o}}\left( \frac{1}{X_{q}}-\frac{1}{X_{d}}\right)U^{2}\sin\left( 2p\gamma_{o}\right)-\frac{3}{2}p\Psi_{o}\frac{\sqrt{2}U}{X_{d}}\cos\left( p\gamma_{o}\right)\qquad \leftarrow (7.13)}\)
Stosujemy podstawienie:
\(\displaystyle{ p\gamma_{o}=\frac{3}{2} \pi -\vartheta \qquad \leftarrow (7.13)}\)
gdzie: \(\displaystyle{ \vartheta}\) - to tzw. kąt mocy.

Wtedy (7.11), (7.12) i (7.13) przyjmują postać:
\(\displaystyle{ I_{q}=\frac{\sqrt{2}U \sin \vartheta}{X_{q}} \qquad \leftarrow (7.14)}\)
\(\displaystyle{ I_{d}=\frac{\sqrt{2}U \cos\vartheta-\omega_{o}\Psi_{o}}{X_{d}} \qquad \leftarrow (7.15)}\)
\(\displaystyle{ T_{e}=\frac{3}{2}\frac{p}{\omega_{o}}\left( \frac{1}{X_{q}}-\frac{1}{X_{d}}\right)U^{2}\sin 2\vartheta+\frac{3}{2}p\Psi_{o}\frac{\sqrt{2}U}{X_{d}}\sin \vartheta \qquad \leftarrow (7.16)}\)
Jak widać nawet przy pominięciu rezystancji uzwojenia silnika PMSM obliczenia stanu ustalonego pracy nie są łatwe. Najpierw na podstawie (7.16) wyznaczamy kąt mocy \(\displaystyle{ \vartheta}\). Potem na podstawie (7.14) i (7.15) obliczamy składowe prądu \(\displaystyle{ I_{d}, I_{q}}\). Następnie obliczamy wartość skuteczną prądu:
\(\displaystyle{ I=\frac{1}{\sqrt{2}}\sqrt{ {I_{d}}^{2}+ {I_{q}}^{2} } \qquad \leftarrow (7.17)}\)
Model wektorowy silnika indukcyjnego.

Równania silnika indukcyjnego we współrzędnych naturalnych:
\(\displaystyle{ \textbf{u}_{s}=\textbf{R}_{s}\textbf{i}_{s}+\frac{d}{dt}\left( \textbf{M}_{ss}\textbf{i}_{s}\right)+\frac{d}{dt}\left( \textbf{M}_{sr}\textbf{i}_{r}\right) \qquad \leftarrow (8.1)}\)
\(\displaystyle{ \textbf{u}_{r}=\textbf{R}_{r}\textbf{i}_{r}+\frac{d}{dt}\left( \textbf{M}_{rr}\textbf{i}_{r}\right)+\frac{d}{dt}\left( \textbf{M}_{rs}\textbf{i}_{s}\right) \qquad \leftarrow (8.2)}\)
gdzie:
\(\displaystyle{ \textbf{R}_{s}=\begin{bmatrix}R_{s}&0&0\\0&R_{s}&0\\0&0&R_{s}\end{bmatrix}, \qquad \textbf{R}_{r}=\begin{bmatrix}R_{r}&0&0\\0&R_{r}&0\\0&0&R_{r}\end{bmatrix} \qquad \leftarrow (8.3)}\)
\(\displaystyle{ \textbf{M}_{ss}=\begin{bmatrix}L_{\sigma s}+M_{s}&-\frac{1}{2}M_{s}&-\frac{1}{2}M_{s}\\-\frac{1}{2}M_{s}&L_{\sigma s}+M_{s}&-\frac{1}{2}M_{s}\\-\frac{1}{2}M_{s}&-\frac{1}{2}M_{s}&L_{\sigma s}+M_{s}\end{bmatrix} \qquad \leftarrow (8.4)}\)
\(\displaystyle{ \textbf{M}_{rr}=\begin{bmatrix}L_{\sigma r}+M_{r}&-\frac{1}{2}M_{r}&-\frac{1}{2}M_{r}\\-\frac{1}{2}M_{r}&L_{\sigma r}+M_{r}&-\frac{1}{2}M_{r}\\-\frac{1}{2}M_{r}&-\frac{1}{2}M_{r}&L_{\sigma r}+M_{r}\end{bmatrix} \qquad \leftarrow (8.5)}\)
\(\displaystyle{ \textbf{M}_{sr}=M_{sr}\begin{bmatrix}\cos p \gamma & \cos \left( p \gamma+120^{\circ}\right) & \cos \left( p\gamma-120^{\circ}\right)\\ \cos \left( p\gamma-120^{\circ}\right) &\cos p \gamma &\cos \left( p \gamma+120^{\circ}\right)\\ \cos \left( p\gamma+120^{\circ}\right) & \cos \left(p \gamma-120^{\circ}\right)& \cos p \gamma \end{bmatrix} \qquad \leftarrow (8.6)}\)
\(\displaystyle{ \textbf{M}_{rs}=\textbf{M}_{sr}^{T} \qquad \leftarrow (8.7)}\)
\(\displaystyle{ \textbf{u}_{s}=\begin{bmatrix}u_{as}\\u_{bs}\\u_{cs}\end{bmatrix}, \quad \textbf{u}_{r}=\begin{bmatrix}u_{ar}\\u_{br}\\u_{cr}\end{bmatrix} \quad \textbf{i}_{s}=\begin{bmatrix}i_{as}\\i_{bs}\\i_{cs}\end{bmatrix}, \quad \textbf{i}_{r}=\begin{bmatrix}i_{ar}\\i_{br}\\i_{cr}\end{bmatrix} \qquad \leftarrow (8.8)}\)
Mnożąc lewostronnie równania (8.1) i (8.2) przez macierz \(\displaystyle{ \textbf{S}}\):
\(\displaystyle{ \textbf{S}\textbf{u}_{s}=\textbf{S}\textbf{R}_{s}\textbf{i}_{s}+\frac{d}{dt}\left( \textbf{S}\textbf{M}_{ss}\textbf{i}_{s}\right)+\frac{d}{dt}\left(\textbf{S} \textbf{M}_{sr}\textbf{i}_{r}\right) \qquad \leftarrow (8.9)}\)
\(\displaystyle{ \textbf{S}\textbf{u}_{r}=\textbf{S}\textbf{R}_{r}\textbf{i}_{r}+\frac{d}{dt}\left(\textbf{S} \textbf{M}_{rr}\textbf{i}_{r}\right)+\frac{d}{dt}\left(\textbf{S} \textbf{M}_{rs}\textbf{i}_{s}\right) \qquad \leftarrow (8.10)}\)
i wyrażając macierze prądów \(\displaystyle{ \textbf{i}_{s}, \textbf{i}_{r}}\) i napięć \(\displaystyle{ \textbf{u}_{s}, \textbf{u}_{r}}\) przez macierz składowych \(\displaystyle{ \underline{0}, I, II}\) (patrz wzór (1.4)):
\(\displaystyle{ \textbf{i}_{s}=\textbf{S}^{-1} \textbf{i}_{s}^{+}, \qquad \textbf{u}_{s}^{+}=\textbf{S} \textbf{u}_{s}, \qquad \textbf{i}_{r}=\textbf{S}^{-1} \textbf{i}_{r}^{+}, \qquad \textbf{u}_{r}^{+}=\textbf{S} \textbf{u}_{r}\qquad \leftarrow (8.11)}\)
otrzymujemy:
\(\displaystyle{ \textbf{u}_{s}^{+}=\textbf{S}\textbf{R}_{s}\textbf{S}^{-1} \textbf{i}_{s}^{+}+\frac{d}{dt}\left( \textbf{S}\textbf{M}_{ss}\textbf{S}^{-1} \textbf{i}_{s}^{+}\right)+\frac{d}{dt}\left(\textbf{S} \textbf{M}_{sr}\textbf{S}^{-1} \textbf{i}_{r}^{+}\right) \qquad \leftarrow (8.12)}\)
\(\displaystyle{ \textbf{u}_{r}^{+}=\textbf{S}\textbf{R}_{r}\textbf{S}^{-1} \textbf{i}_{r}^{+}+\frac{d}{dt}\left(\textbf{S} \textbf{M}_{rr}\textbf{S}^{-1} \textbf{i}_{r}^{+}\right)+\frac{d}{dt}\left(\textbf{S} \textbf{M}_{rs}\textbf{S}^{-1} \textbf{i}_{s}^{+}\right) \qquad \leftarrow (8.13)}\)
Teraz wykonujemy przekształcenia macierzy:    
Po przekształceniu macierzy otrzymujemy równania:
\(\displaystyle{ u_{s}^{\underline{0}}=R_{s}i_{s}^{\underline{0}}+L_{\sigma s}\frac{d i_{s}^{\underline{0}}}{dt} \\ u_{s}^{I}=R_{s}i_{s}^{I}+\left( L_{\sigma s}+\frac{3}{2}M_{s}\right) \frac{d i_{s}^{I}}{dt}+\frac{3}{2}M_{sr}\frac{d}{dt}\left( i_{r}^{I}e^{jp \gamma}\right)\qquad \leftarrow (8.14) \\u_{s}^{II}=R_{s}i_{s}^{II}+\left( L_{\sigma s}+\frac{3}{2}M_{s}\right) \frac{d i_{s}^{II}}{dt}+\frac{3}{2}M_{sr}\frac{d}{dt}\left( i_{r}^{II}e^{-jp \gamma}\right)}\)
\(\displaystyle{ u_{r}^{\underline{0}}=R_{r}i_{r}^{\underline{0}}+L_{\sigma r}\frac{d i_{r}^{\underline{0}}}{dt} \\ u_{r}^{I}=R_{r}i_{r}^{I}+\left( L_{\sigma r}+\frac{3}{2}M_{r}\right) \frac{d i_{r}^{I}}{dt}+\frac{3}{2}M_{sr}\frac{d}{dt}\left( i_{s}^{I}e^{-jp \gamma}\right)\qquad \leftarrow (8.15) \\u_{r}^{II}=R_{r}i_{r}^{II}+\left( L_{\sigma r}+\frac{3}{2}M_{r}\right) \frac{d i_{r}^{II}}{dt}+\frac{3}{2}M_{sr}\frac{d}{dt}\left( i_{s}^{II}e^{jp \gamma}\right)}\)
Najczęściej pomija się równania dla składowej zerowej (z wielu powodów), zaś składowa \(\displaystyle{ II}\) nie wnosi żadnych dodatkowych informacji, bo jest sprzężona do składowej \(\displaystyle{ I}\). Zatem równania silnika indukcyjnego (czy ogólniej: maszyny indukcyjnej, pierścieniowej i klatkowej) są postaci:
\(\displaystyle{ u_{s}^{I}=R_{s}i_{s}^{I}+\left( L_{\sigma s}+\frac{3}{2}M_{s}\right) \frac{d i_{s}^{I}}{dt}+\frac{3}{2}M_{sr}\frac{d}{dt}\left( i_{r}^{I}e^{jp \gamma}\right) \qquad \leftarrow (8.16)\\
u_{r}^{I}=R_{r}i_{r}^{I}+\left( L_{\sigma r}+\frac{3}{2}M_{r}\right) \frac{d i_{r}^{I}}{dt}+\frac{3}{2}M_{sr}\frac{d}{dt}\left( i_{s}^{I}e^{-jp \gamma}\right)\qquad \leftarrow (8.17)}\)
Składową \(\displaystyle{ i_{s}^{I}}\) prądów stojana silnika oznaczymy symbolem wektora przestrzennego \(\displaystyle{ \underline{\textbf{I}}_{s}^{s}}\), przy czym górny indeks \(\displaystyle{ s}\) oznacza to, że wektor ten jest określony w nieruchomym względem stojana układzie współrzędnych.
Podobnie czynimy ze składową \(\displaystyle{ u_{s}^{I}}\) napięć fazowych stojana, tzn. oznaczymy ją symbolem \(\displaystyle{ \underline{\textbf{U}}_{s}^{s}}\).

Składową \(\displaystyle{ i_{r}^{I}}\) prądów wirnika silnika oznaczymy symbolem wektora przestrzennego \(\displaystyle{ \underline{\textbf{I}}_{r}^{r}}\), przy czym górny indeks \(\displaystyle{ r}\) oznacza to, że wektor ten jest określony w nieruchomym względem wirnika układzie współrzędnych.
Analogicznie składową \(\displaystyle{ u_{r}^{I}}\) napięć fazowych wirnika oznaczymy symbolem \(\displaystyle{ \underline{\textbf{U}}_{r}^{r}}\).

Podsumowując:
\(\displaystyle{ i_{s}^{I} \rightarrow \underline{\textbf{I}}_{s}^{s}, \quad u_{s}^{I} \rightarrow \underline{\textbf{U}}_{s}^{s}, \quad i_{r}^{I} \rightarrow \underline{\textbf{I}}_{r}^{r}, \quad u_{r}^{I} \rightarrow \underline{\textbf{U}}_{r}^{r} \qquad \leftarrow (8.18)}\)
Teraz określimy jeden układ współrzędnych, obrócony o kąt \(\displaystyle{ \theta}\) względem układu współrzędnych nieruchomego względem stojana, i wyrazimy wszystkie wektory: \(\displaystyle{ \underline{\textbf{I}}_{s}^{s}, \underline{\textbf{U}}_{s}^{s}, \underline{\textbf{I}}_{r}^{r}, \underline{\textbf{U}}_{r}^{r}}\) w wyżej określonym wspólnym układzie współrzędnych:
\(\displaystyle{ \underline{\textbf{I}}_{s}^{\theta}=\underline{\textbf{I}}_{s}^{s}e^{-j\theta}, \quad \underline{\textbf{U}}_{s}^{\theta}=\underline{\textbf{U}}_{s}^{s}e^{-j\theta}, \quad \underline{\textbf{I}}_{r}^{\theta}=\underline{\textbf{I}}_{r}^{s}e^{-j\theta}, \quad \underline{\textbf{U}}_{r}^{\theta}=\underline{\textbf{U}}_{r}^{s}e^{-j\theta}\qquad \leftarrow (8.19)}\)
przy czym:
\(\displaystyle{ \underline{\textbf{I}}_{r}^{r}=\underline{\textbf{I}}_{r}^{s}e^{-jp\gamma}, \quad \underline{\textbf{U}}_{r}^{r}=\underline{\textbf{U}}_{r}^{s}e^{-jp\gamma}\qquad \leftarrow (8.20)}\)
Na podstawie (8.19) i (8.20) mamy:
\(\displaystyle{ \underline{\textbf{I}}_{s}^{s}=\underline{\textbf{I}}_{s}^{\theta}e^{j\theta}, \quad \underline{\textbf{U}}_{s}^{s}=\underline{\textbf{U}}_{s}^{\theta}e^{j\theta}, \quad \underline{\textbf{I}}_{r}^{r}=\underline{\textbf{I}}_{r}^{\theta}e^{j\left( \theta-p\gamma\right) }, \quad \underline{\textbf{U}}_{r}^{r}=\underline{\textbf{U}}_{r}^{\theta}e^{j\left( \theta-p\gamma\right)}\qquad \leftarrow (8.21)}\)
Na podstawie (8.16), (8.17), (8.18) i (8.21) otrzymujemy:
\(\displaystyle{ \underline{\textbf{U}}_{s}^{\theta}e^{j\theta}=R_{s}\underline{\textbf{I}}_{s}^{\theta}e^{j\theta}+\left( L_{\sigma s}+\frac{3}{2}M_{s}\right) \frac{d}{dt}\left( \underline{\textbf{I}}_{s}^{\theta}e^{j\theta}\right) +\frac{3}{2}M_{sr}\frac{d}{dt}\left( \underline{\textbf{I}}_{r}^{\theta}e^{j\left( \theta-p\gamma\right) }e^{jp \gamma}\right)}\)
\(\displaystyle{ \underline{\textbf{U}}_{r}^{\theta}e^{j\left( \theta-p\gamma\right)}=R_{r}\underline{\textbf{I}}_{r}^{\theta}e^{j\left( \theta-p\gamma\right) }+\left( L_{\sigma r}+\frac{3}{2}M_{r}\right) \frac{d}{dt}\left( \underline{\textbf{I}}_{r}^{\theta}e^{j\left( \theta-p\gamma\right) }\right) +\frac{3}{2}M_{sr}\frac{d}{dt}\left( \underline{\textbf{I}}_{s}^{\theta}e^{j\theta}e^{-jp \gamma}\right)}\)
Po uproszczeniu otrzymujemy:
\(\displaystyle{ \underline{\textbf{U}}_{s}^{\theta}=R_{s}\underline{\textbf{I}}_{s}^{\theta}+\left( L_{\sigma s}+\frac{3}{2}M_{s}\right) \frac{d \underline{\textbf{I}}_{s}^{\theta}}{dt}+j\omega_{\theta}\left( L_{\sigma s}+\frac{3}{2}M_{s}\right) \underline{\textbf{I}}_{s}^{\theta}+\\+\frac{3}{2}M_{sr}\frac{d \underline{\textbf{I}}_{r}^{\theta}}{dt}+j\omega_{\theta}\frac{3}{2}M_{sr}\underline{\textbf{I}}_{r}^{\theta} \qquad \leftarrow (8.22)}\)
\(\displaystyle{ \underline{\textbf{U}}_{r}^{\theta}=R_{r}\underline{\textbf{I}}_{r}^{\theta}+\left( L_{\sigma r}+\frac{3}{2}M_{r}\right) \frac{d \underline{\textbf{I}}_{r}^{\theta}}{dt}+j\left( \omega_{\theta}-p\omega\right) \left( L_{\sigma r}+\frac{3}{2}M_{r}\right) \underline{\textbf{I}}_{r}^{\theta}+\\ +\frac{3}{2}M_{sr}\frac{d \underline{\textbf{I}}_{s}^{\theta}}{dt}+j\left( \omega_{\theta}-p\omega\right) \frac{3}{2}M_{sr}\underline{\textbf{I}}_{s}^{\theta} \qquad \leftarrow (8.23)}\)
\(\displaystyle{ \omega_{\theta}=\frac{d \theta}{dt} \qquad \leftarrow (8.24)}\)
Definiujemy teraz strumienie skojarzone stojana \(\displaystyle{ \mathbf{\underline{\Psi}}_{s}^{\theta}}\) i wirnika \(\displaystyle{ \mathbf{\underline{\Psi}}_{r}^{\theta}}\):

\(\displaystyle{ \mathbf{\underline{\Psi}}_{s}^{\theta}=\left( L_{\sigma s}+\frac{3}{2}M_{s}\right)\underline{\textbf{I}}_{s}^{\theta}+\frac{3}{2}M_{sr} \underline{\textbf{I}}_{r}^{\theta} \qquad \leftarrow (8.25)}\)
\(\displaystyle{ \mathbf{\underline{\Psi}}_{r}^{\theta}=\left( L_{\sigma r}+\frac{3}{2}M_{r}\right)\underline{\textbf{I}}_{r}^{\theta}+\frac{3}{2}M_{sr} \underline{\textbf{I}}_{s}^{\theta} \qquad \leftarrow (8.26)}\)
Wtedy model wektorowy silnika indukcyjnego przyjmuje postać:
\(\displaystyle{ \underline{\textbf{U}}_{s}^{\theta}=R_{s}\underline{\textbf{I}}_{s}^{\theta} +\frac{d \mathbf{\underline{\Psi}}_{s}^{\theta}}{dt}+j\omega_{\theta} \mathbf{\underline{\Psi}}_{s}^{\theta} \qquad \leftarrow (8.27)}\)
\(\displaystyle{ \underline{\textbf{U}}_{r}^{\theta}=R_{r}\underline{\textbf{I}}_{r}^{\theta}+\frac{d \mathbf{\underline{\Psi}}_{r}^{\theta}}{dt}+j\left( \omega_{\theta}-p\omega \right) \mathbf{\underline{\Psi}}_{r}^{\theta} \qquad \leftarrow (8.28)}\)
\(\displaystyle{ \mathbf{\underline{\Psi}}_{s}^{\theta}=L_{s}\underline{\textbf{I}}_{s}^{\theta}+L_{m} \underline{\textbf{I}}_{r}^{\theta}, \qquad\mathbf{\underline{\Psi}}_{r}^{\theta}=L_{r}\underline{\textbf{I}}_{r}^{\theta}+L_{m} \underline{\textbf{I}}_{s}^{\theta} \qquad \leftarrow (8.29)}\)
\(\displaystyle{ L_{s}= L_{\sigma s}+\frac{3}{2}M_{s},\qquad L_{r}= L_{\sigma r}+\frac{3}{2}M_{r},\qquad L_{m}=\frac{3}{2}M_{sr}\qquad \leftarrow (8.30)}\)
\(\displaystyle{ \omega=\frac{d \gamma}{dt},\qquad \omega_{\theta}=\frac{d \theta}{dt}\qquad \leftarrow (8.31)}\)
Przy czym należy pamiętać o zależnościach:
\(\displaystyle{ \underline{\textbf{I}}_{s}^{s}=\underline{\textbf{I}}_{s}^{\theta}e^{j\theta}, \quad \underline{\textbf{U}}_{s}^{s}=\underline{\textbf{U}}_{s}^{\theta}e^{j\theta}, \quad \underline{\textbf{I}}_{r}^{r}=\underline{\textbf{I}}_{r}^{\theta}e^{j\left( \theta-p\gamma\right) }, \quad \underline{\textbf{U}}_{r}^{r}=\underline{\textbf{U}}_{r}^{\theta}e^{j\left( \theta-p\gamma\right)}\qquad \leftarrow (8.32)}\)
Jak widać, model we współrzędnych naturalnych (fazowych) o ośmiu parametrach \(\displaystyle{ R_{s}, R_{r}, L_{\sigma s}, L_{\sigma r}, M_{s}, M_{r}, M_{sr}, p}\) został zastąpiony modelem wektorowym o parametrach: \(\displaystyle{ R_{s}, R_{r}, L_{s}, L_{r}, L_{m}, p}\)

Teraz na podstawie bilansu energetycznego wyznaczymy wyrażenie na moment elektromagnetyczny silnika indukcyjnego. Na podstawie wzoru (1.14), (1.19), (8.27), (8.28) wyznaczamy moc chwilową \(\displaystyle{ P}\) dwóch źródeł napięcia zasilających uzwojenia stojana i wirnika:
\(\displaystyle{ P=\frac{3}{4}u_{s}^{\underline{0}}i_{s}^{\underline{0}}+\frac{3}{4}u_{r}^{\underline{0}}i_{r}^{\underline{0}}+\frac{3}{2} \Re\left\{ \underline{\textbf{U}}_{s}^{\theta} {\cdot \underline{\textbf{I}}_{s}^{\theta}} ^{*}+\underline{\textbf{U}}_{r}^{\theta} \cdot {\underline{\textbf{I}}_{r}^{\theta}}^{*} \right\}=\\
\\
=\frac{3}{2}\Re\left\{ -jp\omega \mathbf{\underline{\Psi}}_{r}^{\theta} \cdot {\underline{\textbf{I}}_{r}^{\theta}}^{*}\right\} + Reszta=\\
\\
=\frac{3}{2}\Re\left\{ -jp\omega \left( L_{r}\underline{\textbf{I}}_{r}^{\theta}+L_{m} \underline{\textbf{I}}_{s}^{\theta} \right) \cdot {\underline{\textbf{I}}_{r}^{\theta}}^{*} \right\} + Reszta=\\
\\
=\frac{3}{2}\Re\left\{ -jp\omega L_{m} \underline{\textbf{I}}_{s}^{\theta} \cdot {\underline{\textbf{I}}_{r}^{\theta}}^{*} \right\} + Reszta=\\
\\
=\frac{3}{2}\Re\left\{ -jp\omega L_{m} \cdot \left[ \Re \left( \underline{\textbf{I}}_{s}^{\theta}{\underline{\textbf{I}}_{r}^{\theta}}^{*}\right) + j \ \Im \left( \underline{\textbf{I}}_{s}^{\theta}{\underline{\textbf{I}}_{r}^{\theta}}^{*}\right) \right] \right\} + Reszta=\\
\\
=\frac{3}{2} \omega p L_{m} \ \Im \left\{ \underline{\textbf{I}}_{s}^{\theta} \ {\underline{\textbf{I}}_{r}^{\theta}}^{*}\right\} + Reszta=}\)
\(\displaystyle{ Reszta}\) - suma wyrażeń, które nie zależą od \(\displaystyle{ \omega}\) w sposób jawny.

Na podstawie powyższego wyrażenie na moment elektromagnetyczny silnika indukcyjnego ma postać:
\(\displaystyle{ T_{e}=\frac{3}{2} p L_{m} \ \Im \left\{ \underline{\textbf{I}}_{s}^{\theta} \cdot {\underline{\textbf{I}}_{r}^{\theta}}^{*}\right\} \qquad \leftarrow (8.33)}\)
Wzór (8.33) można przedstawić w innych postaciach:
\(\displaystyle{ T_{e}=\frac{3}{2} p L_{m} I_{s} I_{r} \cdot \sin \angle\left( \underline{\textbf{I}}_{s}^{\theta},\underline{\textbf{I}}_{r}^{\theta}\right) \qquad \leftarrow (8.34)}\)
Ukryta treść:    
\(\displaystyle{ T_{e}=\frac{3}{2} p \frac{L_{m}}{L_{r}} \cdot \Im\left\{ { \mathbf{\underline{\Psi}}_{r}^{\theta}}^{*} \cdot \underline{\textbf{I}}_{s}^{\theta}\right\} \qquad \leftarrow (8.35)}\)
\(\displaystyle{ T_{e}=\frac{3}{2} p \ \frac{L_{m}}{L_{s}L_{r}-{L_{m}}^{2}} \cdot \Im\left\{ \mathbf{\underline{\Psi}}_{s}^{\theta} \cdot { \mathbf{\underline{\Psi}}_{r}^{\theta}}^{*} \right\} \qquad \leftarrow (8.36)}\)
\(\displaystyle{ T_{e}=\frac{3}{2} p \ \frac{L_{m}}{L_{s}L_{r}-{L_{m}}^{2}} \Psi_{s} \Psi_{r} \cdot \sin \angle\left( \mathbf{\underline{\Psi}}_{s}^{\theta},\mathbf{\underline{\Psi}}_{r}^{\theta}\right) \qquad \leftarrow (8.37)}\)
Ukryta treść:    
\(\displaystyle{ T_{e}=\frac{3}{2} p \ \Im\left\{ { \mathbf{\underline{\Psi}}_{s}^{\theta}}^{*} \cdot \underline{\textbf{I}}_{s}^{\theta}\right\} \qquad \leftarrow (8.38)}\)
Polecam lekturę: SPOSOBY STEROWANIA MOMENTEM W NOWOCZESNYM NAPĘDZIE ELEKTRYCZNYM http://pcc.imir.agh.edu.pl/poz15/poz15.pdf
a także książkę z serii wydawniczej Komitetu Elektrotechniki PAN:
Postępy napędu elektrycznego i energoelektroniki. Tom 48: Teresa Orłowska-Kowalska: Bezczujnikowe układy napędowe z silnikami indukcyjnymi. Oficyna wydawnicza Politechniki Wrocławskiej.
oraz:
Bimal K. Bose: Modern power electronics and AC drives - Prentice Hall PTR (2002) - książka dostępna w internecie.


Sterowanie FOC klatkowego silnika indukcyjnego.

FOC - ang. Field Oriented Control (sterowanie polowo zorientowane).

Przyjęcie układu odniesienia, takiego że: \(\displaystyle{ \Im \mathbf{\underline{\Psi}}_{r}^{\theta} =0}\), a więc można napisać: \(\displaystyle{ \mathbf{\underline{\Psi}}_{r}^{\theta}=\Psi_{r}}\), powoduje, że równania (8.28) i (8.29) przyjmują postać:
\(\displaystyle{ 0=R_{r}\underline{\textbf{I}}_{r}^{\theta}+\frac{d \Psi_{r}}{dt}+j\left( \omega_{\Psi_{r}}-p\omega \right) \Psi_{r}}\)
\(\displaystyle{ \Psi_{r}=L_{r}\underline{\textbf{I}}_{r}^{\theta}+L_{m} \underline{\textbf{I}}_{s}^{\theta}}\)
Wektory prądów \(\displaystyle{ \underline{\textbf{I}}_{r}^{\theta}}\) i \(\displaystyle{ \underline{\textbf{I}}_{s}^{\theta}}\) rozkładamy na części rzeczywiste i urojone, stąd otrzymujemy cztery równania:
\(\displaystyle{ R_{r}I_{rx}+\frac{d \Psi_{r}}{dt}=0\\
R_{r}I_{ry}+\left( \omega_{\Psi r}-p\omega\right)=0\\
L_{r}I_{rx}+L_{m}I_{sx}=\Psi_{r}\\
L_{r}I_{ry}+L_{m}I_{sy}=0}\)
Na podstawie powyższego mamy:
\(\displaystyle{ \omega_{\Psi r}=p\omega+\frac{R_{r}L_{m}}{L_{r}} \frac{I_{sy}}{\Psi_{r}} \qquad \leftarrow (8.39)\\
\frac{d \Psi_{r}}{dt}=\frac{R_{r}}{L_{r}}\left( L_{m}I_{sx}-\Psi_{r}\right) \qquad \leftarrow (8.40)}\)
Wzór na moment, na podstawie powyższego i wzoru (8.35):
\(\displaystyle{ T_{e}=\frac{3}{2}p\frac{L_{m}}{L_{r}} \Psi_{r}I_{sy} \qquad \leftarrow (8.41)}\)
Widać powyżej, że przy \(\displaystyle{ I_{sx}=const.}\) mamy \(\displaystyle{ \Psi_{r}=L_{m}I_{sx}=const.}\). Wtedy to moment silnika jest wprost proporcjonalny do składowej \(\displaystyle{ I_{sy}}\). Idea FOC polega więc na tym, by wyodrębnić z wektora prądu stojana dwie składowe: składową "strumieniową" oraz składową "momentową". Żeby to uczynić należy estymować wektor strumienia wirnika - to jest główna trudność. Niestety nie można tego robić wprost na podstawie opisanego tutaj modelu - choćby dlatego, że "efektywne" rezystancje uzwojeń stojana i wirnika zależą od wielu czynników (np. od temperatury). Trzeba stosować modele adaptacyjne. Stabilizując strumień wirnika za pomocą składowej "strumieniowej" wektora prądu stojana, możemy łatwo wpływać na moment silnika za pomocą składowej "momentowej" wektora prądu stojana.

Jeśli zaś chodzi o strukturę układu regulacji, to wartość zadana składowej "momentowej" wektora prądu stojana wyznaczana jest przez regulator na podstawie uchybu prędkości silnika (która jest mierzona przez enkoder zasprzęglony z wałem silnika), natomiast wartość zadana składowej "strumieniowej" wektora prądu stojana wyznaczana jest przez inny regulator na podstawie uchybu strumienia wirnika. Obliczone wartości zadane obu składowych wektora prądu stojana służą do wyznaczania stanów kluczy tranzystorowych w inwerterze zasilającym silnik.


Sterowanie DTC klatkowego silnika indukcyjnego.


DTC - ang. Direct Torque Control (bezpośrednie sterowanie momentem).

Przyjęcie układu odniesienia, takiego że: \(\displaystyle{ \Im \mathbf{\underline{\Psi}}_{s}^{\theta} =0}\), a więc można napisać: \(\displaystyle{ \mathbf{\underline{\Psi}}_{s}^{\theta}=\Psi_{s}}\), powoduje, że równanie (8.27) przyjmuje postać:
\(\displaystyle{ \underline{\textbf{U}}_{s}^{\theta}=R_{s}\underline{\textbf{I}}_{s}^{\theta} +\frac{d \Psi_{s}}{dt}+j\omega_{\Psi_{s}}\Psi_{s}}\)
Równanie to zapisujemy za pomocą składowych wektorów napięcia i prądu \(\displaystyle{ \underline{\textbf{U}}_{s}^{\theta}=U_{sx}+jU_{sy}; \underline{\textbf{I}}_{s}^{\theta}=I_{sx}+jI_{sy}}\), przez co otrzymujemy dwa równania:
\(\displaystyle{ U_{sx}=R_{s}I_{sx}+\frac{d \Psi_{s}}{dt}\\
U_{sy}=R_{s}I_{sy}+\omega_{\Psi_{s}} \Psi_{s}}\)
Jeśli powyżej zaniedbać składniki \(\displaystyle{ R_{s}I_{sx}, R_{s}I_{sy}}\), to otrzymujemy ważne zależności:
\(\displaystyle{ \frac{d \Psi_{s}}{dt} \approx U_{sx} \qquad \leftarrow (8.42)}\)
\(\displaystyle{ \omega_{\Psi_{s}} \approx \frac{U_{sy}}{\Psi_{s}} \qquad \leftarrow (8.43)}\)
Z zależności (8.42) i (8.43) wnioskujemy, że za pomocą składowych \(\displaystyle{ U_{sx}, U_{sy}}\) wektora napięcia \(\displaystyle{ \underline{\textbf{U}}_{s}}\) można bezpośrednio wpływać na wartość strumienia stojana oraz możemy kontrolować położenie wektora strumienia stojana. Prędkość wirowania wektora strumienia stojana możemy zmieniać skokowo. Również szybkość zmian modułu strumienia stojana można zmieniać skokowo. Pytanie: jaką mamy korzyść z możliwości kontroli położenia wektora strumienia stojana? W tym miejscu należy zwrócić uwagę na wzór (8.37):
\(\displaystyle{ T_{e}=\frac{3}{2} p \ \frac{L_{m}}{L_{s}L_{r}-{L_{m}}^{2}} \Psi_{s} \Psi_{r} \cdot \sin \angle\left( \mathbf{\underline{\Psi}}_{s}^{\theta},\mathbf{\underline{\Psi}}_{r}^{\theta}\right)}\)
Widzimy, że zmieniając kąt pomiędzy wektorami strumieni stojana i wirnika możemy wpływać na moment elektromagnetyczny silnika, ale pod warunkiem, że wektor \(\displaystyle{ \mathbf{\underline{\Psi}}_{r}^{\theta}}\) będzie bardziej "leniwy" niż wektor \(\displaystyle{ \mathbf{\underline{\Psi}}_{s}^{\theta}}\), wtedy kątem pomiędzy tymi wektorami będzie można sterować (zadając odpowiedni wektor napięcia). Okazuje się, że warunek ten jest spełniony. Z czego to wynika? Proszę zerknąć na równania obwodu wirnika, które zostały wyprowadzone przy okazji wyjaśnienia metody FOC:
\(\displaystyle{ \omega_{\Psi r}=p\omega+\frac{R_{r}L_{m}}{L_{r}} \frac{{\red I_{sy}}}{\Psi_{r}} \\ \frac{d \Psi_{r}}{dt}=\frac{R_{r}}{L_{r}}\left( L_{m} {\red I_{sx}} - \Psi_{r}\right)}\)
Co prawda został tam wykorzystany inny układ odniesienia (składowe wektora prądu stojana nie są w obu przypadkach tożsame!), ale można w obu równaniach zauważyć, że strumień wirnika jest wymuszany prądem stojana, zaś prądy stojana nie mogą zmieniać się skokowo. Musi być zachowana ciągłość składowych wektora prądu stojana. Widać więc, że prędkość wirowania \(\displaystyle{ \omega_{\Psi r}}\) nie może zmieniać się tak szybko jak prędkość \(\displaystyle{ \omega_{\Psi_{s}}}\), którą można zmieniać skokowo. Jeśli chodzi o szybkość zmian modułu wektora strumienia wirnika \(\displaystyle{ \Psi_{r}}\), to również i tutaj mamy zdecydowanie wolniejsze zmiany niż w przypadku strumienia \(\displaystyle{ \Psi_{s}}\) - bo mamy powyżej równanie różniczkowe pierwszego rzędu ze względu na \(\displaystyle{ \Psi_{r}}\), w którym wymuszeniem jest prąd stojana ("dopiero" prąd stojana, a nie napięcie stojana - stąd ta bezwładność wektora strumienia wirnika). Wektor strumienia stojana reaguje bezpośrednio na napięcie stojana, zaś wektor strumienia wirnika reaguje "dopiero" (!) na prąd stojana, a nie bezpośrednio na napięcie stojana. Dzięki temu DTC działa. Podobnie jak w przypadku FOC potrzebny jest dobry model adaptacyjny silnika, który pozwoli na estymację wektora strumienia stojana oraz wartości momentu silnika. Podczas regulacji momentu w metodzie DTC staramy się stabilizować strumień stojana po to, aby silnik był przez cały czas namagnesowany w takim samym stopniu (podobnie w metodzie FOC stabilizowany jest strumień wirnika).

Przyjrzyjmy się zachowaniu wektorów strumieni nieco dokładniej. Przypomnijmy wcześniej uzyskany wynik. Prędkość \(\displaystyle{ \omega_{x}}\) wirowania wektora przestrzennego \(\displaystyle{ \underline{\textbf{X}}}\):
\(\displaystyle{ \omega_{x}=\frac{1}{X^{2}}\Im \left\{ \dot{\underline{\textbf{X}}} \cdot {\underline{\textbf{X}}}^{*} \right\} \qquad \leftarrow (8.44)}\)
zaś szybkość zmiany modułu tego wektora:
\(\displaystyle{ \frac{dX}{dt}=\frac{1}{X} \Re \left\{ \dot{\underline{\textbf{X}}} \cdot {\underline{\textbf{X}}}^{*} \right\} \qquad \leftarrow (8.45)}\)
Na podstawie (8.29) można napisać zależność odwrotną:
\(\displaystyle{ \underline{\textbf{I}}_{s}^{\theta}=\frac{1}{\Delta}\left( L_{r}\mathbf{\underline{\Psi}}_{s}^{\theta}-L_{m}\mathbf{\underline{\Psi}}_{r}^{\theta}\right), \qquad \underline{\textbf{I}}_{r}^{\theta}=\frac{1}{\Delta}\left( L_{s}\mathbf{\underline{\Psi}}_{r}^{\theta}-L_{m}\mathbf{\underline{\Psi}}_{s}^{\theta}\right) \qquad \leftarrow (8.46)}\)
\(\displaystyle{ \Delta=L_{s}L_{r}-{L_{m}}^{2} \qquad \leftarrow (8.47)}\)
Na podstawie równań (8.27), (8.28) modelu silnika oraz na podstawie (8.46), po przyjęciu nieruchomego względem stojana układu odniesienia, tj. dla \(\displaystyle{ \omega_{\theta}=0}\), otrzymujemy:
\(\displaystyle{ \frac{d \mathbf{\underline{\Psi}}_{s}^{\theta}}{dt} =\underline{\textbf{U}}_{s}^{\theta} - R_{s}\underline{\textbf{I}}_{s}^{\theta}=\underline{\textbf{U}}_{s}^{\theta} - \frac{R_{s}}{\Delta}\left( L_{r}\mathbf{\underline{\Psi}}_{s}^{\theta}-L_{m}\mathbf{\underline{\Psi}}_{r}^{\theta}\right) \qquad \leftarrow (8.48)}\)
\(\displaystyle{ \frac{d \mathbf{\underline{\Psi}}_{r}^{\theta}}{dt}=jp\omega \mathbf{\underline{\Psi}}_{r}^{\theta}-R_{r}\underline{\textbf{I}}_{r}^{\theta}=jp\omega \mathbf{\underline{\Psi}}_{r}^{\theta}-\frac{R_{r}}{\Delta}\left( L_{s}\mathbf{\underline{\Psi}}_{r}^{\theta}-L_{m}\mathbf{\underline{\Psi}}_{s}^{\theta}\right) \qquad \leftarrow (8.49)}\)
Na podstawie wzoru (8.44) prędkości wirowania \(\displaystyle{ \omega_{\Psi_{s}}, \omega_{\Psi_{r}}}\) wektorów strumienia stojana i wirnika \(\displaystyle{ \mathbf{\underline{\Psi}}_{s}^{\theta},\mathbf{\underline{\Psi}}_{r}^{\theta}}\)::
\(\displaystyle{ \omega_{\Psi_{s}} = \frac{1}{{\Psi_{s}}^{2}}\Im \left\{ \frac{d \mathbf{\underline{\Psi}}_{s}^{\theta}}{dt} \cdot {\mathbf{\underline{\Psi}}_{s}^{\theta}}^{*} \right\} \qquad \leftarrow (8.50)}\)
\(\displaystyle{ \omega_{\Psi_{r}} = \frac{1}{{\Psi_{r}}^{2}}\Im \left\{ \frac{d \mathbf{\underline{\Psi}}_{r}^{\theta}}{dt} \cdot {\mathbf{\underline{\Psi}}_{r}^{\theta}}^{*} \right\} \qquad \leftarrow (8.51)}\)
Na podstawie wzoru (8.45) szybkość zmian modułów wektorów strumienia stojana i wirnika \(\displaystyle{ \mathbf{\underline{\Psi}}_{s}^{\theta},\mathbf{\underline{\Psi}}_{r}^{\theta}}\)::
\(\displaystyle{ \frac{d \Psi_{s}}{dt} = \frac{1}{\Psi_{s}}\Re \left\{ \frac{d \mathbf{\underline{\Psi}}_{s}^{\theta}}{dt} \cdot {\mathbf{\underline{\Psi}}_{s}^{\theta}}^{*} \right\} \qquad \leftarrow (8.52)}\)
\(\displaystyle{ \frac{d \Psi_{r}}{dt} = \frac{1}{\Psi_{r}}\Re \left\{ \frac{d \mathbf{\underline{\Psi}}_{r}^{\theta}}{dt} \cdot {\mathbf{\underline{\Psi}}_{r}^{\theta}}^{*} \right\} \qquad \leftarrow (8.53)}\)

Teraz obliczamy następujące wyrażenia:
\(\displaystyle{ \frac{d \mathbf{\underline{\Psi}}_{s}^{\theta}}{dt} \cdot {\mathbf{\underline{\Psi}}_{s}^{\theta}}^{*}=\underline{\textbf{U}}_{s}^{\theta} \cdot {\mathbf{\underline{\Psi}}_{s}^{\theta}}^{*}- \frac{R_{s}L_{r}}{\Delta} {\Psi_{s}}^{2}+\frac{R_{s}L_{m}}{\Delta}\mathbf{\underline{\Psi}}_{r}^{\theta} \cdot {\mathbf{\underline{\Psi}}_{s}^{\theta}}^{*} \qquad \leftarrow (8.54)}\)
\(\displaystyle{ \frac{d \mathbf{\underline{\Psi}}_{r}^{\theta}}{dt} \cdot {\mathbf{\underline{\Psi}}_{r}^{\theta}}^{*}=jp\omega \cdot {\Psi_{r}}^{2}- \frac{R_{r}L_{s}}{\Delta} {\Psi_{r}}^{2}+\frac{R_{r}L_{m}}{\Delta}\mathbf{\underline{\Psi}}_{s}^{\theta} \cdot {\mathbf{\underline{\Psi}}_{r}^{\theta}}^{*} \qquad \leftarrow (8.55)}\)
Ostatecznie prędkości wirowania wektorów strumieni stojana i wirnika, oraz szybkość zmian modułów tych wektorów (na podstawie (8.50 \(\displaystyle{ \div}\) 8.55)):
\(\displaystyle{ \omega_{\Psi_{s}}=\frac{U_{s}}{\Psi_{s}}\sin \angle \left( \underline{\textbf{U}}_{s}^{\theta}, \mathbf{\underline{\Psi}}_{s}^{\theta}\right)-\frac{R_{s}L_{m}}{\Delta} \frac{\Psi_{r}}{\Psi_{s}}\sin \angle \left(\mathbf{\underline{\Psi}}_{s}^{\theta}, \mathbf{\underline{\Psi}}_{r}^{\theta}\right) \qquad \leftarrow (8.56) \\
\\
\\
\omega_{\Psi_{r}}=p\omega+\frac{R_{r}L_{m}}{\Delta}\frac{\Psi_{s}}{\Psi_{r}}\sin \angle \left(\mathbf{\underline{\Psi}}_{s}^{\theta}, \mathbf{\underline{\Psi}}_{r}^{\theta}\right) \qquad \leftarrow (8.57) \\
\\
\\
\frac{d \Psi_{s}}{dt}=U_{s} \cos \angle \left( \underline{\textbf{U}}_{s}^{\theta}, \mathbf{\underline{\Psi}}_{s}^{\theta}\right)+\frac{R_{s}L_{m}}{\Delta} \Psi_{r}\cos \angle \left(\mathbf{\underline{\Psi}}_{s}^{\theta}, \mathbf{\underline{\Psi}}_{r}^{\theta}\right) - \frac{R_{s}L_{r}}{\Delta} \Psi_{s} \qquad \leftarrow (8.58) \\
\\
\\
\frac{d \Psi_{r}}{dt}=\frac{R_{r}L_{m}}{\Delta} \Psi_{s} \cos \angle \left(\mathbf{\underline{\Psi}}_{s}^{\theta}, \mathbf{\underline{\Psi}}_{r}^{\theta}\right)- \frac{R_{r}L_{s}}{\Delta} \Psi_{r} \qquad \leftarrow (8.59)}\)
Wnioski:
1) Tylko \(\displaystyle{ \omega_{\Psi_{s}}}\) i \(\displaystyle{ \frac{d \Psi_{s}}{dt}}\) mogą zmieniać się skokowo, o ile można zmieniać skokowo wektor napięcia stojana \(\displaystyle{ \underline{\textbf{U}}_{s}^{\theta}}\).
2) Jeśli będziemy stabilizować strumień \(\displaystyle{ \Psi_{s}}\), to również stabilizowany będzie strumień \(\displaystyle{ \Psi_{r}}\) (wynika to z równania (8.59)).


Klasyczny schemat zastępczy silnika indukcyjnego.


Analiza dotycząca indukcyjności uzwojeń silnika: \(\displaystyle{ M_{s}, M_{r},L_{\sigma s}, L_{\sigma r}, M_{sr}}\) prowadzi do wniosku, że nie wszystkie wyżej wymienione indukcyjności są niezależne. Dochodzi się do związków następujących:
\(\displaystyle{ M_{r}=\frac{M_{s}}{\nu^2}, \qquad M_{sr}=\frac{M_{s}}{\nu} \qquad \leftarrow (8.39)}\)
gdzie: \(\displaystyle{ \nu}\) - przekładnia uwzględniająca liczby zwojów i rozkład uzwojeń silnika.
Ciąg dalszy nastąpi.


Wektory przestrzenne a składowe symetryczne.


Przypomnijmy sobie teraz podstawowe zależności dotyczące analizy harmonicznej. Załóżmy, że sygnał \(\displaystyle{ x(t)}\) ma przebieg Fouriera:
\(\displaystyle{ \begin{aligned} x(t)&=X_{o}+\sum_{n=1}^{ \infty} a_{n}\cos \left(n \omega_{o}t\right) +\sum_{n=1}^{ \infty} b_{n}\sin \left( n \omega_{o}t\right)= \\& =X_{o}+\sum_{n=1}^{ \infty} c_{n}\cos \left(n \omega_{o}t+\psi_{n}\right) \qquad \leftarrow (10.1) \end{aligned}}\)
\(\displaystyle{ \omega_{o}=\frac{2 \pi }{T}, \quad X_{o}=\frac{1}{T}\int\limits_{t_{o}}^{t_{o}+T} x(t)dt \qquad \leftarrow (10.2)}\)
\(\displaystyle{ a_{n}=\frac{2}{T}\int\limits_{t_{o}}^{t_{o}+T} x(t)\cos \left( n\omega_{o}t\right)dt, \quad b_{n}=\frac{2}{T}\int\limits_{t_{o}}^{t_{o}+T} x(t)\sin \left( n\omega_{o}t\right)dt \qquad \leftarrow (10.3)}\)
\(\displaystyle{ c_{n}=\sqrt{{a_{n}}^{2}+{b_{n}}^{2}}, \quad c_{n}\cos\psi_{n}=a_{n}, \quad -c_{n}\sin\psi_{n}=b_{n} \qquad \leftarrow (10.4)}\)
Szereg Fouriera (10.1) można przedstawić w równoważnej postaci:
\(\displaystyle{ x(t)=X_{o}+\sqrt{2} \ \Re \sum_{n=1}^{ \infty} \underline{X}_{n} e^{j n \omega_{o}t} \qquad \leftarrow (10.5)}\)
\(\displaystyle{ \underline{X}_{n}=\frac{c_{n}}{\sqrt{2}}e^{j\psi_{n}}=\frac{a_{n}-jb_{n}}{\sqrt{2}}=X_{n}e^{j\psi_{n}}=\frac{\sqrt{2}}{T}\int\limits_{t_{o}}^{t_{o}+T}x(t)e^{-jn\omega_{o}t}dt \qquad \leftarrow (10.6)}\)
Mając dwa sygnały \(\displaystyle{ x(t), y(t)}\):
\(\displaystyle{ x(t)=X_{o}+\sqrt{2} \ \Re \sum_{n=1}^{ \infty} \underline{X}_{n} e^{j n \omega_{o}t}, \quad y(t)=Y_{o}+\sqrt{2} \ \Re \sum_{n=1}^{ \infty} \underline{Y}_{n} e^{j n \omega_{o}t}}\)
możemy obliczyć ich iloczyn skalarny:
\(\displaystyle{ \langle x,y \rangle=\frac{1}{T}\int\limits_{t_{o}}^{t_{o}+T} x(t)y(t)dt=X_{o}Y_{o}+\Re\sum_{n=1}^{ \infty } \underline{X}_{n}\underline{Y}_{n}^{*}\qquad \leftarrow (10.7)}\)
oraz wartości skuteczne:
\(\displaystyle{ \|x\|=\sqrt{\langle x,x \rangle}={X_{o}}^{2}+\sum_{n=1}^{ \infty }{X_{n}}^{2} \qquad \leftarrow (10.8)}\)

Wpływ harmonicznych na zachowanie wektora przestrzennego.

Przyjmijmy trzy zmienne fazowe \(\displaystyle{ x_{A}, x_{B}, x_{C}}\):
\(\displaystyle{ x_{A}(t)=X_{Ao}+\sqrt{2} \ \Re \sum_{n=1}^{ \infty} \underline{X}_{An} e^{j n \omega_{o}t} \\
x_{B}(t)=X_{Bo}+\sqrt{2} \ \Re \sum_{n=1}^{ \infty} \underline{X}_{Bn} e^{j n \omega_{o}t} \qquad \leftarrow (10.9)\\
x_{C}(t)=X_{Co}+\sqrt{2} \ \Re \sum_{n=1}^{ \infty} \underline{X}_{Cn} e^{j n \omega_{o}t}}\)
Korzystając z ogólnej zależności: \(\displaystyle{ \cos \gamma=\frac{1}{2}\left(e^{j\gamma}+e^{-j\gamma}\right)}\), (10.9) można zapisać jeszcze inaczej:
\(\displaystyle{ x_{A}(t)=X_{Ao}+\frac{\sqrt{2}}{2} \sum_{n=1}^{ \infty} \left[ \underline{X}_{An} e^{j n \omega_{o}t}+\underline{X}_{An}^{*} e^{-j n \omega_{o}t}\right]\\
x_{B}(t)=X_{Bo}+\frac{\sqrt{2}}{2} \sum_{n=1}^{ \infty} \left[ \underline{X}_{Bn} e^{j n \omega_{o}t}+\underline{X}_{Bn}^{*} e^{-j n \omega_{o}t}\right] \qquad \leftarrow (10.10)\\
x_{C}(t)=X_{Co}+\frac{\sqrt{2}}{2} \sum_{n=1}^{ \infty} \left[ \underline{X}_{Cn} e^{j n \omega_{o}t}+\underline{X}_{Cn}^{*} e^{-j n \omega_{o}t}\right]}\)
Wtedy wektor przestrzenny \(\displaystyle{ \underline{\textbf{X}}}\) oraz składowa zerowa \(\displaystyle{ X_{0}}\) reprezentujące zmienne fazowe \(\displaystyle{ x_{A}, x_{B}, x_{C}}\):
\(\displaystyle{ \begin{aligned}X_{0}(t)=\frac{1}{3}\left( x_{A}+x_{B}+x_{C}\right)=X_{0o}+\sqrt{2} \ \Re \sum_{n=1}^{ \infty} \underline{X}_{0n} e^{j n \omega_{o}t} \end{aligned}\qquad \leftarrow (10.11)}\)
\(\displaystyle{ \begin{aligned}X_{0o}&=\frac{1}{3}\left( X_{Ao}+X_{Bo}+X_{Co}\right) \\ \underline{X}_{0n}=\frac{1}{3}\left( \underline{X}_{An}+\underline{X}_{Bn}+\underline{X}_{Cn}\right)\end{aligned}\qquad \leftarrow (10.12)}\)
\(\displaystyle{ \begin{aligned}\underline{\textbf{X}}(t)=&\frac{2}{3}\left(x_{A}+a \cdot x_{B}+a^{2} \cdot x_{C}\right)=\\[2ex]=&\frac{2}{3}\left( X_{Ao}+aX_{Bo}+a^{2}X_{Co}\right)+
\\[2ex]&+\frac{\sqrt{2}}{3} \sum_{n=1}^{ \infty} \left( \underline{X}_{An}+a\underline{X}_{Bn}+a^{2}\underline{X}_{Cn}\right) e^{jn \omega_{o}t} +\\[2ex]&+\frac{\sqrt{2}}{3} \sum_{n=1}^{ \infty} \left( \underline{X}_{An}^{*}+a\underline{X}_{Bn}^{*}+a^{2}\underline{X}_{Cn}^{*}\right) e^{-jn \omega_{o}t} \end{aligned} \qquad \leftarrow (10.13)}\)
Ukryta treść:    
\(\displaystyle{ \begin{aligned} X_{\alpha}(t)&=\frac{1}{3}\left( 2x_{A}-x_{B}-x_{C}\right)=X_{\alpha o}+\sqrt{2} \ \Re \sum_{n=1}^{ \infty} \underline{X}_{\alpha n} e^{j n \omega_{o}t}\\
X_{\beta}(t)&=\frac{1}{\sqrt{3}}\left( x_{B}-x_{C}\right)=X_{\beta o}+\sqrt{2} \ \Re \sum_{n=1}^{ \infty} \underline{X}_{\beta n} e^{j n \omega_{o}t} \end{aligned} \qquad \leftarrow (10.14)}\)
\(\displaystyle{ \begin{aligned}X_{\alpha o}&=\frac{1}{3}\left( 2X_{Ao}-X_{Bo}-X_{Co}\right)\\X_{\beta o}&=\frac{1}{\sqrt{3}}\left( X_{Bo}-X_{Co}\right) \end{aligned} \qquad \leftarrow (10.15)}\)
\(\displaystyle{ \begin{aligned}\underline{X}_{\alpha n}&=\frac{1}{3}\left( 2\underline{X}_{An}-\underline{X}_{Bn}-\underline{X}_{Cn}\right)\\\underline{X}_{\beta n}&=\frac{1}{\sqrt{3}}\left( \underline{X}_{Bn}-\underline{X}_{Cn}\right) \end{aligned} \qquad \leftarrow (10.16)}\)
\(\displaystyle{ \begin{aligned} X_{\alpha}(t)&=X_{\alpha o}+\frac{\sqrt{2}}{2} \sum_{n=1}^{ \infty} \left[ \underline{X}_{\alpha n} e^{jn \omega_{o}t}+\underline{X}_{\alpha n}^{*}e^{-j n \omega_{o}t }\right] \end{aligned}\qquad \leftarrow (10.17)}\)
\(\displaystyle{ \begin{aligned} X_{\beta}(t)&=X_{\beta o}+\frac{\sqrt{2}}{2} \sum_{n=1}^{ \infty} \left[ \underline{X}_{\beta n} e^{jn \omega_{o}t}+\underline{X}_{\beta n}^{*}e^{-j n \omega_{o}t }\right] \end{aligned}\qquad \leftarrow (10.18)}\)
Wracamy na chwilę do zależności (10.11), (10.12), (10.13). Widać w nich ślad pojęcia tzw. składowych symetrycznych, które występują w teorii trójfazowych obwodów elektrycznych. Można te zależności przepisać następująco:
\(\displaystyle{ \begin{aligned}X_{0}(t)=&X_{0o}+\sqrt{2} \ \Re \sum_{n=1}^{ \infty} \underline{X}^{(0)}_{n} e^{j n \omega_{o}t} \\
\underline{\textbf{X}}(t)=&\underline{\textbf{X}}_{o}+\sqrt{2} \sum_{n=1}^{ \infty} \underline{X}^{(1)}_{n} e^{jn \omega_{o}t} +\sqrt{2} \sum_{n=1}^{ \infty} \left[ \underline{X}^{(2)}_{n}\right] ^{*} e^{-jn \omega_{o}t} \end{aligned}\qquad \leftarrow (10.19)}\)
gdzie:
\(\displaystyle{ \begin{aligned}\underline{\textbf{X}}_{o}=&\frac{2}{3}\left( X_{Ao}+a \ X_{Bo}+a^{2}X_{Co}\right) \\
\underline{X}^{(0)}_{n}=&\frac{1}{3}\left( \underline{X}_{An}+\underline{X}_{Bn}+\underline{X}_{Cn}\right) \\
\underline{X}^{(1)}_{n}=&\frac{1}{3}\left( \underline{X}_{An}+a \ \underline{X}_{Bn}+a^{2}\underline{X}_{Cn}\right)\\
\underline{X}^{(2)}_{n}=&\frac{1}{3}\left( \underline{X}_{An}+a^{2} \underline{X}_{Bn}+a \ \underline{X}_{Cn}\right)\end{aligned}\qquad \leftarrow (10.20)}\)
\(\displaystyle{ \underline{\textbf{X}}_{o}}\) - składowa stała wektora przestrzennego \(\displaystyle{ \underline{\textbf{X}}}\)

\(\displaystyle{ \underline{X}^{(0)}_{n}}\) - składowa symetryczna zerowa harmonicznych rzędu \(\displaystyle{ n}\) zmiennych fazowych \(\displaystyle{ x_{A}, x_{B}, x_{C}}\);

\(\displaystyle{ \underline{X}^{(1)}_{n}}\) - składowa symetryczna zgodna harmonicznych rzędu \(\displaystyle{ n}\) zmiennych fazowych \(\displaystyle{ x_{A}, x_{B}, x_{C}}\);

\(\displaystyle{ \underline{X}^{(2)}_{n}}\) - składowa symetryczna przeciwna harmonicznych rzędu \(\displaystyle{ n}\) zmiennych fazowych \(\displaystyle{ x_{A}, x_{B}, x_{C}}\).

Transformacja Edyty Clarke a Transformacja Parka.

Dotychczas przedstawiona została Transformacja Edyty Clarke. Przedstawiona tutaj "konstrukcja" tej transformacji opiera się na idei wektora przestrzennego z 1959 roku. Pierwotnie transformacja Edyty Clarke została sformułowana znacznie wcześniej niż idea wektora przestrzennego.

Napiszmy równania (6.10) transformacji Edyty Clarke w postaci macierzowej:
\(\displaystyle{ \begin{bmatrix}x_{\alpha}\\x_{\beta}\\x_{0}\end{bmatrix} =\frac{2}{3} \begin{bmatrix}1&-\frac{1}{2}&-\frac{1}{2}\\0&\frac{\sqrt{3}}{2}&-\frac{\sqrt{3}}{2}\\\frac{1}{2}&\frac{1}{2}&\frac{1}{2}\end{bmatrix} \cdot \begin{bmatrix}x_{A}\\x_{B}\\x_{C}\end{bmatrix}\qquad \leftarrow (11.1)}\)
Równanie to można przedstawić w łatwiejszej do zapamiętania postaci:
\(\displaystyle{ \begin{bmatrix}x_{\alpha}\\x_{\beta}\\x_{0}\end{bmatrix} =\frac{1}{3} \begin{bmatrix}2&-1&-1\\0&\sqrt{3}&-\sqrt{3}\\ 1& 1& 1\end{bmatrix} \cdot \begin{bmatrix}x_{A}\\x_{B}\\x_{C}\end{bmatrix}\qquad \leftarrow (11.2)}\)
Zaś transformacja odwrotna:
\(\displaystyle{ \begin{bmatrix}x_{A}\\x_{B}\\x_{C}\end{bmatrix} = \begin{bmatrix}1&0&1\\-\frac{1}{2}&\frac{\sqrt{3}}{2}&1\\ -\frac{1}{2}&-\frac{\sqrt{3}}{2}&1\end{bmatrix} \cdot \begin{bmatrix}x_{\alpha}\\x_{\beta}\\x_{0}\end{bmatrix}\qquad \leftarrow (11.3)}\)
Transformację Parka (https://en.wikipedia.org/wiki/Direct-quadrature-zero_transformation) można skonstruować w następujący sposób. Wyrażamy współrzędne wektora przestrzennego \(\displaystyle{ \underline{\textbf{X}}_{\alpha\beta}}\) w wirującym układzie współrzędnych:
\(\displaystyle{ \begin{aligned} \underline{\textbf{X}}_{dq}&=x_{d}+jx_{q}=\underline{\textbf{X}}_{\alpha\beta}e^{-j\theta}=\\&=\left( x_{\alpha}+jx_{\beta}\right) \cdot \left( \cos\theta-j\sin \theta\right) =\\&=\left( x_{\alpha}\cos \theta+x_{\beta} \sin \theta\right) +j\left(x_{\beta} \cos \theta- x_{\alpha}\sin \theta\right)\end{aligned} \qquad \leftarrow (11.4)}\)
Czyli transformujemy współrzędne \(\displaystyle{ \alpha, \beta}\) do współrzędnych \(\displaystyle{ d,q}\). Na podstawie (11.4) można napisać:
\(\displaystyle{ \begin{aligned}x_{d}=x_{\alpha}\cos \theta+x_{\beta} \sin \theta \\ x_{q}= x_{\beta} \cos \theta- x_{\alpha}\sin \theta\end{aligned} \qquad \leftarrow (11.5)}\)
Współrzędnej 0 (składowej zerowej) nie zmieniamy.
\(\displaystyle{ \begin{bmatrix}x_{d}\\x_{q}\\x_{0}\end{bmatrix} = \begin{bmatrix}\cos\theta&\sin\theta&0\\-\sin\theta&\cos\theta&0\\ 0&0&1\end{bmatrix} \cdot \begin{bmatrix}x_{\alpha}\\x_{\beta}\\x_{0}\end{bmatrix}\qquad \leftarrow (11.6)}\)
Oznaczmy macierz transformacji Edyty Clarke przez \(\displaystyle{ T_{C}}\), a macierz transformacji w powyższym przekształceniu (11.6) przez \(\displaystyle{ T_{P}}\):
\(\displaystyle{ T_{C}=\frac{2}{3} \begin{bmatrix}1&-\frac{1}{2}&-\frac{1}{2}\\0&\frac{\sqrt{3}}{2}&-\frac{\sqrt{3}}{2}\\\frac{1}{2}&\frac{1}{2}&\frac{1}{2}\end{bmatrix}, \qquad T_{P}=\begin{bmatrix}\cos\theta&\sin\theta&0\\-\sin\theta&\cos\theta&0\\ 0&0&1\end{bmatrix} \qquad \leftarrow (11.7)}\)
\(\displaystyle{ {T_{C}}^{-1}=\begin{bmatrix}1&0&1\\-\frac{1}{2}&\frac{\sqrt{3}}{2}&1\\ -\frac{1}{2}&-\frac{\sqrt{3}}{2}&1\end{bmatrix} , \qquad {T_{P}}^{-1}=\begin{bmatrix}\cos\theta&-\sin\theta&0\\ \sin\theta&\cos\theta&0\\ 0&0&1\end{bmatrix} \qquad \leftarrow (11.8)}\)
Złożenie obu przekształceń prowadzi do transformacji Parka (Robert H. Park https://en.wikipedia.org/wiki/Robert_H._Park):
\(\displaystyle{ \begin{bmatrix}x_{d}\\x_{q}\\x_{0}\end{bmatrix}=T_{CP}\begin{bmatrix}x_{A}\\x_{B}\\x_{C}\end{bmatrix} \qquad \leftarrow (11.9)}\)
gdzie:
\(\displaystyle{ T_{CP}=T_{P} \cdot T_{C} \qquad \leftarrow (11.10)}\)
\(\displaystyle{ T_{CP}=\frac{2}{3}\begin{bmatrix}\cos\left( \theta\right) &\cos\left( \theta-120^{\circ}\right) &\cos\left( \theta+120^{\circ}\right)\\ -\sin\left( \theta\right) &-\sin\left( \theta-120^{\circ}\right) &-\sin\left( \theta+120^{\circ}\right)\\ \frac{1}{2}&\frac{1}{2}&\frac{1}{2}\end{bmatrix} \qquad \leftarrow (11.11)}\)
\(\displaystyle{ {T_{CP}}^{-1}=\begin{bmatrix}\cos\left( \theta\right) &-\sin\left( \theta\right) &1\\ \cos\left( \theta-120^{\circ}\right) &-\sin\left( \theta-120^{\circ}\right) &1\\
\cos\left( \theta+120^{\circ}\right)&-\sin\left( \theta+120^{\circ}\right)&1\end{bmatrix} \qquad \leftarrow (11.12)}\)

Park, Inverse Park and Clarke, Inverse Clarke Transformations MSS Software Implementation. User Guide. https://www.microsemi.com/document-portal/doc_view/132799-park-inverse-park-and-clarke-inverse-clarke-transformations-mss-software-implementation-user-guide

Clarke and Park Transforms on the TMS320C2xx. http://www.ti.com/lit/an/bpra048/bpra048.pdf

Koniecznie do przeczytania: Clarke’s and Park’s Transformations http://read.pudn.com/downloads169/sourcecode/embed/779893/Part%204%20Clark&Park.pdf

R. H. Park, "Two-reaction theory of synchronous machines - generalized method of analysis - Part 1", AIEE Trans., vol. 48, pp. 716-730, July 1929. https://uwaterloo.ca/power-energy-systems-group/sites/ca.power-energy-systems-group/files/uploads/files/two-reaction.pdf

dq Transformations http://www.sze.hu/~szenasy/PM%20DC%20jegyz/riaz/dqtranformeq.pdf


Ciąg dalszy nastąpi.
ODPOWIEDZ